You are on page 1of 110

MODULE FOR CLASSICAL

THE UNIVERSITY OF ZAMBIA MECHANICS

CLASSICAL MECHANICS
PHY 2511

UNIVERSITY OF ZAMBIA
SCHOOL OF NATURAL SCIENCES
©Copyright

Reccab Ochieng Manyala (2016)

This module has specifically been written for distance education students of the University of Zambia.
The program may use the module in all reasonable ways. However, the copyright remains with the
author. No part of this module may be reproduced or transmitted in any form or by any means,
electronic or mechanical, including photocopying, recording or by any information storage and
retrieval system, without permission in writing from the author.

RECCAB OCHIENG MANYALA


SCHOOL OF NATURAL SCIENCES, DEPARTMENT OF PHYSICS.
Acknowledgement

I would like to sincerely thank Dr. Mweene for his acceptance to allow me use material from his
module for Fast Track Teacher Education Programme. Without his good gesture, this module could
not have been completed in time as some of the materials presented here were directly taken from his
module which already has a structure for a course in Classical Mechanics, Analytical Mechanics and
Special Theory of Relativity for a programme such as the one for distance education.
CLASSICAL MECHANICS

Contents
Contents i

About this Module 1


How this Module is structured .......................................................................................... 1

Module overview 3
Welcome to Classical Mechanics Module (PHY 2511) ................................................... 3
CLASSICAL MECHANICS Module. Is this module for you? ........................................ 3
Timeframe ......................................................................................................................... 4
Study skills ........................................................................................................................ 4
Need help? ........................................................................................................................ 6
Assignments ...................................................................................................................... 6
Assessments ...................................................................................................................... 7

Getting around this Module 8


Margin icons ..................................................................................................................... 8

Chapter 1 10

Unit 1 Vectors and their Applications 10


1.0 Introduction .................................................................................................. 11
1.1 The Cartesian Coordinate System ................................................................ 11
1.2 The Plane Polar Coordinate System ............................................................ 14
1.3 The Cylindrical Polar Coordinate System ................................................... 20
1.4 The Spherical Polar Coordinate System ...................................................... 23
1.5 Summary ...................................................................................................... 28
1.6 Exercises ...................................................................................................... 28

Chapter 2 30

Unit 2 Particle Dynamics in One Dimension 30


2.0 Introduction .................................................................................................. 31
2.1 Motion of a Particle in One Dimension ....................................................... 33
2.1.1 The Free Particle .......................................................................................... 33
2.1.2 Particle Moving Under the Effect of a Constant Force ................................ 34
2.1.3 The Simple Harmonic Oscillator .................................................................. 36
2.2 General Treatment of Motion in One Dimension ........................................ 38
2.2.1 Time and Velocity Dependent Forces .......................................................... 44
ii Contents

2.2.2 Time-Dependent Forces ............................................................................... 44


2.2.3 Velocity-Dependent Forces .......................................................................... 46
2.3 Summary ...................................................................................................... 47
2.4 Exercises ...................................................................................................... 47

Chapter 3 50

Unit 3 Particle Dynamics in Two and Three Dimensions 50


3.0 Introduction .................................................................................................. 51
3.1 General Theory ............................................................................................ 51
3.2 Particle in the Field of Gravity..................................................................... 53
3.3 Force and Work............................................................................................ 56
3.4 Force and Work............................................................................................ 56
3.5 Force and Work (The Del Operator) ............................................................ 58
3.6 Angular Momentum ..................................................................................... 62
3.7 Central Force Motion ................................................................................... 64
3.8 Summary ...................................................................................................... 69
3.9 Exercises ...................................................................................................... 69

Chapter 4 72

Unit 4 Simple Harmonic Motion 72


4.0 Introduction .................................................................................................. 73
4.1 Displacement in Simple Harmonic Motion ................................................. 74
4.2 Velocity and Acceleration in Simple Harmonic Motion ............................. 78
4.3 The Energy of a Harmonic Oscillator .......................................................... 78
4.4 Simple Harmonic Motion in Nature ............................................................ 84
4.5 Rotary Motion and Simple Harmonic Motion ............................................. 88
4.6 Damped Oscillations .................................................................................... 91
4.7 Energy Dissipation in Damped Motion ....................................................... 98
4.8 Summary .................................................................................................... 100
4.8 Exercises .................................................................................................... 101
CLASSICAL MECHANICS

About this Module


This module on Classical Mechanics has been structured as outlined
below.

How this Module is structured


The course overview

The course overview gives you a general introduction to the course.


Information contained in the course overview will help you determine:

 If the course is suitable for you.

 What you will already need to know.

 What you can expect from the course.

 How much time you will need to invest to complete the course.

The overview also provides guidance on:

 Study skills.

 Where to get help.

 Course assignments and assessments.

 Activity icons.

 Units.

We strongly recommend that you read the overview carefully before


starting your study.

The course content

The course is broken down into units. Each unit comprises:

 An introduction to the unit content.


Module overview

 Unit outcomes.

 New terminology.

 Core content of the unit with a variety of learning activities.

 A unit summary.

 Assignments and/or assessments, as applicable.

Resources

For those interested in learning more on this subject, we provide you with
a list of additional resources at the end of this Module; these may be
books, articles or web sites.

Your comments

After completing this module on Classical Mechanics we would


appreciate it if you would take a few moments to give us your feedback
on any aspect of this course. Your feedback might include comments on:

 Course content and structure. (Have we left out something important?


Or is the material too bulky?)

 Course reading materials and resources.(Did you find these adequate


and useful?)

 Course assignments. (Did you find the concepts in the assignments


consistent with what is given in the module? Did you feel
comfortable solving the problems?

 Course assessments. (Is the weighting in the assessment reasonable in


terms of marks distribution)

 Course duration. (Is the duration sufficient?)

 Course support (assigned tutors, technical help, etc.)

Your constructive feedback will help us to improve and enhance this


course.

2
CLASSICAL MECHANICS

Module overview

7.

Welcome to Classical Mechanics


Module (PHY 2511)
This Module is one of the modules in second year Physics for student
taking the Distance Education Science program in which Physics as one
of their subjects of study at the University of Zambia.

CLASSICAL MECHANICS Module.


Is this module for you?
This course is intended for people who are pursuing a degree in Science
Education and studying Physics as one of the subjects.

The prerequisite for this course is PHY 1010 (Introduction to Physics). It


is, however, advisable that apart from PHY 1010, the learner should have
a good understanding of Mathematics covering topics such as Differential
Equations ( first and second order), Partial Differential Equations,
Complex numbers, Integral Equations, Matrices and Trigonometry.

3
Module overview

Timeframe
This is the module for Classical Mechanics that you will study during your programme.
The other module in the field of Mechanics is PHY 2522-called Analytical Mechanics
and the Special Theory of Relativity. Both modules should be completed in the second
year of study.
The modules cover material (work) for one academic year. For both in each unit, the
time frame required to study the material adequately is indicated. The time indicated is
How long? not the time for lectures but time for self study. Lectures may take less or more time
than indicated. It is advisable to study PHY 2511 first before proceeding to PHY 2522,
however, there are units in PHY 2522 which can be studied independent of those units in
the PHY 2511 module

Study skills

As an adult learner your approach to learning will be different to that


from your school days: you will choose what you want to study, you will
have professional and/or personal motivation for doing so and you will
most likely be fitting your study activities around other professional or
domestic responsibilities.

Essentially you will be taking control of your learning environment. As a


consequence, you will need to consider performance issues related to
time management, goal setting, stress management, etc. Perhaps you will
also need to reacquaint yourself in areas such as essay planning, coping
with exams and using the web as a learning resource.

Your most significant considerations will be time and space i.e. the time
you dedicate to your learning and the environment in which you engage
in that learning.

We recommend that you take time now—before starting your self-


study—to familiarize yourself with these issues. There are a number of
excellent resources on the web. A few suggested links are:

 http://www.how-to-study.com/

The “How to study” web site is dedicated to study skills


resources. You will find links to study preparation (a list of nine

4
CLASSICAL MECHANICS

essentials for a good study place), taking notes, strategies for


reading text books, using reference sources, test anxiety.

 http://www.ucc.vt.edu/stdysk/stdyhlp.html

This is the web site of the Virginia Tech, Division of Student


Affairs. You will find links to time scheduling (including a
“where does time go?” link), a study skill checklist, basic
concentration techniques, control of the study environment, note
taking, how to read essays for analysis, memory skills
(“remembering”).

 http://www.howtostudy.org/resources.php

Another “How to study” web site with useful links to time


management, efficient reading, questioning/listening/observing
skills, getting the most out of doing (“hands-on” learning),
memory building, tips for staying motivated, developing a
learning plan.

The above links are our suggestions to start you on your way. At the time
of writing these web links were active. If you want to look for more go to
www.google.com and type “self-study basics”, “self-study tips”, “self-
study skills” or similar. A part from the given web site addresses, there is
a free book at bookboon.com entitled-“Strategies to Fight Exam Stress
and Achieve Success” ISBN 978-87-7681-917-0 by Will Stringer. This
is an excellent book which gives you guidance on many aspects of
preparing for an exam.

5
Module overview

Need help?

You may find some resources on other website or contact Prof. Reccab
Ochieng Manyala at the Department of Physics, School of Natural
Sciences, University of Zambia. E-mail address: reccabo@yahoo.com.
Help
for matters related to the course. You are also free to consult with any
academic member of staff in the Department of Physics; they are always
willing to help students. In case I am not the one taking you in this
course, you will consult with the lecturer concerned. Though we have
endeavored to make this unit “mistake free” as much as possible, there
could be some mistakes. We encourage you to point out these mistakes
to your course mates so that they do not get the wrong ideas. Please also
point out the mistakes to the lecturer concerned during the course.

Assignments

You will be required to write and submit assignments based on this


module. Details of these assignments will be given to you during the
residential school. Be sure to write these assignments because they have
a bearing on your continuous assessment grades.
Assignments
During the residential school, you will be directed where to submit all
written assignments.

All queries or correspondence concerning your studies to be directed to


the following address:

The Director,

Directorate of Distance Education

University of Zambia

6
CLASSICAL MECHANICS

P.O BOX 32379,

LUSAKA

Assessments

There are a number of assessment items in this module. You will be


required to carry out experiments in the laboratory and write laboratory
reports that shall be graded. At the same time you will be assigned
Assessments
questions (tutorials) that you will be told to attempt and hand in at
appropriate scheduled times. The questions will also be graded. During
the course a minimum of two written tests will be administered and
graded. Finally, you will be required to write a final examination in the
course. The weighting of these assessment items will be as follows:

Assessment item Weight Total

1. Two written tests 20% 20%

2. Laboratory reports 15% 15%

3. Tutorials 5% 5%

4. Final examination 60% 60%

Total 100% 100%

Note. In the case where only one test is given, the test will account for
20%.

Please ensure that you avail yourself for all these assessment items
because they will account for your final grade.

7
8

Getting around this Module

Margin icons
While working through this module you will notice the frequent use of
margin icons. These icons serve to “signpost” a particular piece of text, a
new task or change in activity; they have been included to help you to
find your way around this Module.

A complete icon set is shown below. We suggest that you familiarize


yourself with the icons and their meaning before starting your study.

This module has been written with you as a learner in mind. On the left
margins of each page, reasonable spaces have been left. This is not for
the beauty of the module but the spaces are for you to interact with the
module. You can carry out calculations in the spaces provided, you can
make short note in these spaces or even make comments and put
reminders to help you in your study of this course.

8
CLASSICAL MECHANICS

Activity Assessment Assignment Case study

Discussion Group activity Help Note it!

Outcomes Reading Reflection Study skills

Summary Terminology Time Tip

9
10

Chapter 1
Unit 1 Vectors and their Applications

You are supposed to spend a minimum of 10 hrs to complete this unit. The
10 hrs does not include time for the exercises which you are advised to do
at your own pace in a reasonable manner

OBJECTIVES

(1) The first objective of this unit is to introduce the learner to the coordinate systems that are
most common in the solution of problems in Physics.

(2) Teach the learner how to transform coordinate systems.

After studying this unit, the learner shall be able to:

(1) Differentiate between the coordinate systems that are commonly used in solution of physics
problems.

(2) Transform from one coordinate system to another.

(3) Recognise and identify the correct and appropriate coordinate system to use for a particular
problem.

(4) Solve problems in different coordinate systems.

10
CLASSICAL MECHANICS

1.0 Introduction
To solve many problems in Mechanics or Physics for that matter, it is
often necessary to be able to specify the position of its constituents. This
is a very important stage in the search for a solution because the wrong
choice of coordinates may render the problem difficult or not solvable at
all. The position of the constituents of a system requires the use of
specific coordinate system. We shall make a formal study of the different
options available to be able to solve problems in this module. The four
most commonly used coordinate systems are: the Cartesian or
rectangular coordinate system, the Plane polar coordinate system, the
Cylindrical coordinate system and the Spherical polar coordinate system.

1.1 The Cartesian Coordinate System


The Cartesian coordinate system is the most widely used of all the
coordinate systems. This is a system we encounter in our everyday lives.
For example when we are walking, we normally walk in a straight line
and we may label that direction x . During the walk, we may want to
change direction by making a ninety degree turn. When this is done we
may label that new direction y . On the other hand we may want to jump
up. Jumping up is in a direction perpendicular to both x and y . These
movements complete the description of the Cartesian coordinate system.
Many Physics problems are framed in this coordinate system. A
complete Cartesian coordinate system is illustrated in Fig. 1. It is
constructed as follows. We first choose an origin O to which we attach
three coordinate axes labeled x , y and z which are mutually
orthogonal (perpendicular). The directed line from the origin to the point
P where a particle is located is called the position vector r . The
position of a point such as P is specified by 3 numbers or coordinates
which are the lengths of the components of the vector r , that is x , y
and z .

11
12

P x, y, z 

z
Y

Fig.1: Cartesian coordinates

Each of the coordinate axes has a unit vector which gives its direction.

These unit vectors are respectively iˆ , ĵ and k̂ for the x , y and z


axes. As the names imply, each of these unit vectors has a magnitude of
unity (one). The dot product of any two arbitrary vectors A and B is
defined as

A  B  A B sin  AB sin (1.1)

where  is the angle between the vectors. Since the unit vectors are

mutually orthogonal (perpendicular, that is at 900 ) to each other, they


obey the following relationships:

iˆ  ˆj  iˆ  kˆ  ˆj  kˆ  0 (1.2)

Again for any two vectors A and B their cross product defined as:

A  B  A B cos  AB cos  C (1.3)

results in a third vector C which lies perpendicular to the plane


containing A and B . It is therefore possible to obtain any one of the

12
CLASSICAL MECHANICS

unit vectors as a cross product of the other two. Using a coordinate


system, the formulas are:

iˆ  ˆj  kˆ, ˆj  kˆ  iˆ and kˆ  iˆ  ˆj (1.4)

In terms of the unit vectors, the position vector is

r  iˆx  ˆjy  kˆz  0 (1.5)

An alternative of writing this is

r  x, y, z  (1.6)

The length of a vector is called its magnitude. For the position vector the
magnitude is given by


r  r  r  r  x2  y 2  z 2 1
2
(1.7)

For a moving point P , the coordinates change with time and are therefore
functions of time. The velocity of the point P is the time derivative of
the position vector and is therefore given by

dr ˆ dx ˆ dy ˆ dz
v i  j k (1.8)
dt dt dt dt

It is usual to indicate the time derivative of a quantity by a dot above


quantity in question. This notation allows us to write Eq.(1.8) as

v  r  iˆx  ˆjy  kˆz (1.9)

The magnitude of the velocity, that is, the speed is


v  v  v  v  x 2  y 2  z 2  1
2
(1.10)

Differentiating the velocity with respect to time gives us the acceleration;


this is

a  v  r  iˆx  ˆjy  kˆz (1.11)

13
14

Example 1.1: The position of a particle at a point P in Cartesian

coordinate system is given in metres by the vector r  6iˆ  5 ˆj  2kˆ .


What is the distance of the particle from the origin?

Solution: The required quantity is the magnitude of r and is given by

r  r  62  (5) 2  22  8.06 m

Example 1.2: The position vector of a particle whose mass is 4 kg


moves in space described by a vector that depends on time

     
as r  4t  6t 2 iˆ  t 3  2 ˆj  3t  5t 2 kˆ . Calculate for t  2 s the
position, the velocity and the magnitude of the force acting on the
particle.

Solution: To obtain the position vector at t  2 s , we substitute 2 for t

in the expression for r . This yields r  16iˆ  6 ˆj  26kˆ . The velocity


is given by

v  r  iˆx  ˆjy  kˆz

 
 iˆ4 12t   ˆj 3t 2  kˆ3  10t 

so at t  2 s , v  20iˆ  12 ˆj  23kˆ

According to Newton’s second law, the force is given by ma . Since

a  12iˆ  6tˆj  10kˆ . We have at t  2 s , a  12iˆ  12 ˆj  10kˆ so

that the force is F  ma  48iˆ  48 ˆj  10kˆ . The magnitude of this


force is


F  F   48  482  102
2
 1
2
 68.6 N

1.2 The Plane Polar Coordinate System


This is a very useful coordinate system for solving problems in curved or
circular motion in two dimensions. To construct this system; we start be
defining an origin O , and from that origin draw a horizontal reference
14
CLASSICAL MECHANICS

line labeled X which extends to the negative side. From the origin again
draw a vertical reference line labeled Y and extending in both positive
and negative directions (see Fig. 2).

Y P

O X

Fig. 2: Plane polar coordinates

The position of the point P is measured from the origin defined by the
parameters r (the length from O to P) and the angle  which the line OP
makes with the horizontal reference line in the counter-clockwise
direction. The coordinates of P are then given as r ,   . It is possible to
obtain transformation equations which connect the plane polar
coordinates and the Cartesian coordinates using Fig. 3. The reference line
coincides with the positive x  axis. By simple trigonometry, we
establish that

x  r cos (1.12)

y  r sin (1.13)

The inverse transformations are


r  x2  y 2 
1
2
(1.14)

and

 y  x   y 
  arctan   cos1    sin 1   (1.15)
x  x2  y2   x2  y2 
   
15
16

with the range of coordinates being 0  r   and 0    2 .

Fig.3: Unit vectors r̂ and ˆ in plane polar coordinates

In terms of the Cartesian unit vectors iˆ and ĵ , the radial unit vector is

rˆ  iˆ cos  ˆj sin (1.16)

It is also clear that

ˆ  iˆ sin  ˆj cos (1.17)

We can also relate these unit vectors using Fig. 4.

   
Fig.4: Relation between unit vectors r̂ , ˆ and iˆ, ˆj .

16
CLASSICAL MECHANICS

Using the two equations (1.16) and (1.17) it is possible to verify

that rˆ  ˆ  0 . From these equations, we can solve for the Cartesian unit
vectors to obtain

iˆ  rˆ cos  ˆ sin (1.18)

and

ˆj  rˆ sin  ˆ cos (1.19)

The position vector of the point P can be written as

r  rrˆ (1.20)

and its velocity if therefore

d
r  (rrˆ)  rrˆ  rrˆ (1.21)
dt

As P moves in time,  changes and the direction which ˆ points also


changes. Therefore, unlike the Cartesian unit vectors which are fixed and

whose time derivatives are consequently zero, r̂ and its counterpart ˆ


have non-vanishing time derivatives. The time derivative of the radial
unit vector is

d
rˆ  (iˆ cos  ˆj sin )
dt
 iˆ sin  ˆj cos
 (iˆ sin  ˆj cos )

 ˆ (1.22)

Hence

r  rrˆ  rˆ (1.23)

which shows that velocity in plane polar coordinates has two


components:

vr  r (1.24)

and
17
18

v  r (1.25)

These are respectively called the radial and angular components of the
velocity and are mutually orthogonal. The radial component vr is the

component along r̂ whereas v is the velocity component along ˆ .

The square of the velocity is

v2  v  v  (rrˆ  rˆ)  (rrˆ  rˆ)  r2  r 22 (1.26)

so that the magnitude of the velocity is


v  r 2  r 2 2 
1
2
(1.27)

The acceleration of the system is given by

a
dv dr d

dt dt dt
 
rrˆ  rˆ 
(1.28)
dr drˆ d dr  ˆ d ˆ dˆ d
 rˆ  r    r   r
dt d dt dt dt dt dt


 rrˆ  r ˆ   rˆ  rˆ  r rˆ

That is,

  
a  r  r rˆ  r  2r ˆ  (1.29)

Thus the two components of the acceleration a are the radial


acceleration ar and the angular acceleration a given by

ar  r  r 2 (1.30)

a  r  2r (1.31)

Using equation (1.25) we can write the term

2
v  v2
r 2  r      (1.32)
r  r

which is called the centripetal acceleration arising from the motion in the
 direction. Furthermore, if r is held constant in time then, r  r  0

18
CLASSICAL MECHANICS

and the path is a circle with centripetal acceleration ar   r  v / r .


2 2

The term 2 r is the so called Coriolis acceleration.

Example 1.3: What are the polar coordinates of the point (2, 7) ?


Solution: Using equation (14), we find r  (2)2  72 1
2
=7.28. From

 7 
Eq. (15) we find   arctan   105.9 . Therefore
0

2
(r ,  )  (7.28, 105.9).

Note: When the tangent is negative, the required angle could be in the
second or fourth quadrant. The correct quadrant is determined by which
component of the position vector is giving rise to the negative sign. If it
is the x component as in this case, the angle is in the second quadrant. If
it is the y coordinate, the angle is in the fourth quadrant.

Example 1.4: The position vector of a particle is given by


r  iˆb sin t  ˆjb cost , where b and  are constants. What are its
plane polar coordinates? Determine its acceleration in plane polar
coordinates and describe the motion of the particle.

Solution: The polar coordinates are


r  x2  y 2 1
2

 b 
1
2
sin 2 t  b 2 cos2 t 2

b

and

  t
Description of the motion: the particle is moving on the circumference of
a circle of radius b centred on the origin. The angular velocity of the
particle is constant value  . The acceleration is

19
20

   
a  r  r rˆ  r  2r ˆ
 b 2 rˆ  b 2 rˆ

where we have used r  b  0 and     0 . The particle is clearly

moving under the centripetal acceleration b 2 .

1.3 The Cylindrical Polar Coordinate System


This is a three-dimensional coordinate system constructed by adding a z
axis at right angles to the plane polar coordinate system that we studied in
the last section. The system is shown in Fig. 5.

Fig. 5: Cylindrical coordinates  ,  , z  and the corresponding unit

 
vectors ˆ , ˆ, ẑ .

The unit vectors are in the directions of the increasing vectors associated

with them. It is important to note that zˆ  kˆ is constant, while the unit

vectors ̂ and ˆ are functions of  as in the case of plane polar

20
CLASSICAL MECHANICS

coordinates. The relation between the coordinates ( x, y, z ) and the


cylindrical coordinates are: (see Fig. 5)

x   cos (1.33)

y   sin  (1.34)

zz (1.35)

while the inverse relations are

 x 2  y 2  2
1
(1.36)

 y  y  x 
  tan 1    sin 1    cos1   (1.37)
 x  x2  y 2   x2  y2 
   

The unit vectors in this system are ˆ , ˆ and k̂ . Here ̂ is the same as

r̂ from the previous section, while ˆ is the same as ˆ . These unit


vectors are mutually orthogonal, and so obey

ˆ  ˆ  ˆ  kˆ  ˆ  kˆ  0 (1.38)

Replacing ( r ,  ) by  ,   and with an additional Z component, we


may write the new unit vectors in terms of the Cartesian unit vectors as

ˆ  iˆ cos  ˆj sin (1.39)

ˆ  iˆ sin   ˆj cos  (1.40)

kˆ  kˆ or ẑ  ẑ (1.41)

The coordinate system is right-handed, so that

ˆ  ˆ  kˆ, ˆ  kˆ  ˆ , kˆ  ˆ  ˆ (1.42)

From Eqs. (1.39) and (1.40), we have

dˆ ˆ dˆ
  and   ˆ (1.43)
d d

21
22

In cylindrical coordinate system, the unit vectors ˆ and ˆ move with


the position vector and are therefore implicitly functions of time.

However, the unit vector kˆ  zˆ is constant. The position vector of the


point P can therefore be represented by

r  ˆ  kˆz (1.44)

where  is the distance of P from the Z-axis and  gives its angular
rotation from the X-axis, while z gives its elevation above the XY plane.
Thus we may write the velocity vector, keeping in mind that ˆ  ˆ ( ) ,
as

v  r 
d
dt

ˆ  kˆz 
d dˆ d dz ˆ dkˆ
 ˆ    kz
dt d dt dt dt
 ˆ   (ˆ)  zkˆ

dkˆ
where  0.
dt

Hence,

v  ˆ  ˆ  zkˆ (1.45)

Using Eq. (1.23) from the previous section with r replaced by  and 

replaced by  , we see that

 
v2  v  v  ˆ  ˆ  zkˆ  ˆ  ˆ  zkˆ 
  2   22  z 2 (1.46)

and


v   2   22  z 2 
1
2
(1.47)

Similarly, the acceleration

a
dv d

dt dt

ˆ  ˆ  zkˆ 
22
CLASSICAL MECHANICS

can be shown using Eq. (43) to be

   
a    2 ˆ    2 ˆ  zkˆ (1.48)

Example 1.5: A bead slides on a wire bent into the form of a helix. The
motion of the bead is given in cylindrical coordinates by
  b,   t , z  ct , where b,  , and c are constants.
Determine the velocity and the acceleration of the bead.

Solution:

In this problem   b  0,   ,   0, z  c, z  0

Using Eqs. (1.45) and (1.48), we find that

v  bˆ  ckˆ  bˆ  czˆ

and

a  b 2 ˆ

1.4 The Spherical Polar Coordinate System


We now use our knowledge of the previous sections to build up the
spherical polar coordinate system. Things will now be easy because we
have developed all the tools we require for this work. Spherical polar
coordinates or spherical coordinates are the most commonly used
coordinates in situations where spherical symmetry-for example, in the
case coulomb forces in atoms and gravitational forces. The coordinate
system is illustrated in Fig. 6. It is formed by adding a third axis to the
plane polar coordinate system, but with the component of the position
vector along this axis measured by means of an angle  which gives the
inclination vector to this axis. The coordinates of a point P are
r,  ,  . The position vector r has a length r and its z component is
given by

z  r cos (1.49)

23
24

As shown in Fig.6, r is called the radial distance from the origin O,  is


the azimuthal angle locating a plane whose angle of rotation is measured
from the X-axis, while  is the polar angle measured down from the Z-
axis. The polar angle  can have any value between 0 and  / 2 , while
the azimuthal angle  can have any value between 0 and  . These
limits restrict the description or motion of point P to the first half of the
sphere on the upper plane. It is not uncommon to extend the description
to include the whole sphere.

Fig. 6: (a) Spherical polar coordinates r ,  ,   and the corresponding

   
unit vectors r̂ , ˆ, ˆ . (b) Orientation of unit vectors r̂ , ˆ, ˆ relative

to the coordinate system XYZ and polar angle  .

The x and y components are visualized as follows. We imagine a light


shining straight down the Z-axis so that it casts a shadow of the position
vector in the XY-plane. This shadow is clearly the same as the quantity
 of the cylindrical coordinate system. In terms of this shadow
therefore, the x and y coordinates are

x   cos (1.50)

24
CLASSICAL MECHANICS

y   sin  (1.51)

Clearly from the cylindrical coordinate system or Fig. 6, we note that

  r sin (1.52)

Substituting this into Eq. (1.50) and (1.51) yield

x  r sin cos (1.53)

y  r sin sin  (1.54)

The inverse relations are


r  x2  y 2  z 2 
1
2
(1.55)

Now, according to Fig.6,

 2  x2  y 2 (1.56)

Using Eqs. (1.49) and (1.52), we observe that


tan   (1.57)
z

Therefore,

 x2  y 2 
  tan 1   (1.58)
 z 
 

Using Eqs. (1.53) and (1.54) we obtain

y
  tan 1 (1.59)
x

The three mutually perpendicular unit vectors used in spherical

coordinates are r̂ , ˆ, and ˆ as shown in Fig. 6 (a) and (b). Also

shown are the unit vectors iˆ, ˆj , and zˆ (  kˆ ) , and ̂ . The unit

vector ˆ lies in the XY plane, while rˆ, ˆ, ˆ , and zˆ all lie in one
vertical plane. In terms of the Cartesian unit vectors, the unit vectors of

25
26

the spherical polar coordinate system are as follows. The radial unit
vector is

rˆ  sin cosiˆ  sin sinˆj  coskˆ (1.60)

For the unit vector in the  direction, the z component is easily deduced
to be  sin . To obtain the x and y coordinates of this unit vector, we
shine a light on it from the top parallel to the Z axis. The shadow of the

unit vector ˆ in the XY plane has a length cos so that its x and y
components are cos cos and cos sin respectively. As a result

ˆ  cos cosiˆ  cos sinˆj  sinkˆ (1.61)

From the previous two sections and Fig.6, we see that

ˆ   siniˆ  cosˆj (1.62)

The unit vectors are mutually orthogonal and therefore obey the
following rules

rˆ  ˆ  rˆ  ˆ  ˆ  ˆ  0 (1.63)

and as the coordinate system is right-handed,

ˆ  rˆ  ˆ, rˆ  ˆ  ˆ, ˆ  ˆ  rˆ (1.64)

Differentiating Eqs. (1.60)-(1.62), we obtain the following relations:

rˆ ˆ rˆ ˆ
   sin
 
ˆ ˆ ˆ
  rˆ   cos (1.65)
 
ˆ ˆ
0   ˆ   rˆ sin  ˆ cos
 

These relations can also be derived from geometrical considerations by


drawing figures similar to the ones in the case of plane polar coordinates.

In spherical coordinates, the position vector of a point P in space is given


by the position vector r :

r  rrˆ  rrˆ( ,  ) (1.66)

26
CLASSICAL MECHANICS

We can now find expressions for velocity and acceleration by making use
of the preceding relations. Thus

drˆ drˆ
v  r  (rrˆ)  rrˆ( ,  )  rˆ  r
d d dr
 rrˆ  r (1.67)
dt dt dt dt dt

Using Eq. (1.65),

drˆ drˆ d drˆ d ˆ  ˆ


( ,  )       sin (1.68)
dt d dt d dt

Hence we obtain

v  rrˆ  rˆ  (r sin )ˆ (1.69)

Similarly,

a  v  r 
dv d

dt dt

rrˆ  rˆ  (r sin )ˆ  (1.70)

which on simplification yields

a  (r  r 2  r sin 2 2 )rˆ


 (r  2r  r sin cos2 )ˆ (1.71)
 (r sin  2r sin  2r cos )ˆ

Example 1.6. The position vector of a particle is


r  brˆ,   B sin t ,   ct

where b, B,  , and c are constants. Determine the velocity and


acceleration of the particle. Give a complete description of the trajectory
of the particle.

Solution: Since in general r  rrˆ , it follows that r  b . Therefore

r  b  0,   B cost,    2 B sint,   c,   0

Inserting these values into Eqs. (1.69) and (1.71), we obtain

v  bB costˆ  bc sin(B sint )ˆ

and

27
28

a  (b 2 B 2 cos2 t  bc2 sin2 ( B sin t ))rˆ


 (b 2 B sin t  bc2 sin( B sin t ) cos(B sin t ))ˆ
 2bcB cost cos(B cost ) ˆ

Description of trajectory: The particle moves on the surface of the sphere


of radius b thus keeping a fixed distance from the origin. From

  ct , we see that its motion in the XY plane is a rotation about the Z-


axis with constant angular velocity   c . At the same time the polar
angle changes as if the particle is performing simple harmonic motion in
the plane instantaneously defined the position vector and the Z-axis.
Viewed from the top, the particle seems as if it is a pendulum of
amplitude B and angular frequency  whose plane of swing is

rotating with angular velocity   c .

1.5 Summary
In this unit we have familiarized ourselves with some of the most
common coordinate systems used to solve problems in Physics. There
are a number of other coordinate systems, but they are generally used at
higher level. A good mastery of the particular coordinate systems treated
in this unit will be sufficient for any problems you as the learner will
encounter in this course.

1.6 Exercises
1.0 A honey bee homes in its hive in a spiral path in such a way that
the radial distance decreases at a constant rate, so that r  b  ct , while

the angular speed increases at the constant rate   kt , where b , c and


k are constants. Find the speed of the bee as a function of time.

28
CLASSICAL MECHANICS

1.1 On a horizontal turntable rotating with constant angular velocity,


an insect crawls outward on a radial line in such a way that its radial
distance increases quadratically with time, so that

r  bt 2 and   t , where b and  are constants. Determine the


acceleration of the insect.

1.2 A racing car moves in a circle of radius b . If the speed of the


car varies with time t according to the equation v  ct , where c is a
positive constant, show that the angle between the velocity vector and the

acceleration vector is 450 at time t  b / c .

1.3 The position vector of a certain particle as a function of time is

r (t )  iˆ(1  ekt )  ˆjekt

where k is a positive constant. Find the velocity and the acceleration of


the particle and sketch its trajectory.

1.4 An ant crawls on the surface of a ball of radius b in such a way


that the position of the ant is given by the spherical polar coordinate

 1 
r  b,   t ,  1  cos 4t 
2 4 

Determine the speed of the ant as a function of time and describe the sort
of path represented by this equation.

1.5 The position vector of a particle is given by


r  iˆa sin t  ˆjb cost , where a , b and  are constants. By
eliminating t , show that the path of the particle is an ellipse. Obtain the
speed of the particle.

1.6 Find the force acting on a particle of mass 3 kg at t  3 s if it

has a velocity in m/s given by v  iˆ2t 3  ˆj (1  t 2 ) .

29
30

Chapter 2
Unit 2 Particle Dynamics in One Dimension

You should be able to cover this unit in 20 hrs. This time period does not
include the time for solving problems in the exercises. You may solve the
exercises at your own convenient pace.

OBJECTIVES

(1) To introduce the learner to the fundamentals of dynamics and kinematics

(2) To teach the learner how to solve one-dimensional problems involving position and velocity
dependent forces.

(3) To teach the learner how to solve the dynamical problem in two and three dimensions.

(4) To acquaint the learner with the properties of motion under a conservative force.

(5) To introduce the learner to the properties of central force motion.

After studying this unit, the learner should be able to:

(1) Differentiate between a kinematic and dynamic problem.

(2) Solve one-dimensional problems involving position dependent forces

30
CLASSICAL MECHANICS

(3) Solve dynamical problems in two and three dimensions.

(4) Identify central force problems.

(5) Differential between conservative and non-conservative forces.

(6) Solve problems involving conservative forces

(7) Identify and enumerate the properties of central force motion.

2.0 Introduction
The fundamental problem of mechanics is to determine how a particle
moves under the action of a particular force. For example, the motion of
a particle of mass m along a straight line which we may consider to be
the x  axis under the direction of a force directed along the x  axis ,
constitutes a one dimensional problem. Newton’s second law is the
fundamental equation for treating problems in dynamics.

According to Newton’s second law, if a force F acts on a particle of


mass m , the effect is to impart to the particle an acceleration a given by

F
a (2.1)
m

Newton’s second law (Eq. 2.1), leads in two or three dimensions to the
vector equation. In Cartesian coordinates, two dimension equations is
equivalent to two component equations and in three dimensions to three
component equations.

These are

d 2x d2y d 2z
m  Fx , m  Fy , m  Fz (2.2)
dt 2 dt 2 dt 2

These are differential equations of second order in time, and the


dynamical problems reduce to solving these equations. It is usual in
dynamics to indicate the time derivative by placing a dot over the
quantity being differentiated and so we can write the vector equation as

31
32

d 2r
m  F  mr (2.3)
dt 2

When the force F is specified, this is called the equation of motion. It is


clear that if the force is zero, the equation of motion is

mr  0 (2.4)

In other words

dv
m 0 (2.5)
dt

Integrating Eq. (2.5) once with respect to time t , we deduce that

mv  C (2.6)

where C is a constant. Since mv is the momentum, the constant has the


dimensions of momentum and we denote it by p0 . This allows us to

write

mv  p0 (2.7)

Eq. (2.7) tells us that the linear momentum p  mv is constant when the
force is zero. The equation is therefore a statement of the law of
conservation of linear momentum and states that: The linear momentum
of a particle which is isolated so that no force acts on it is conserved.

Let us divide Eq. (2.6) by m to obtain

dr
v  p0 / m  v (2.8)
dt

From Eq. (2.8) we can write

dr  v0 dt (2.9)

Eq. (2.9) can be integrated one more to obtain

r  v0t  D (2.10)

where D is a constant of integration with the dimensions of


displacement. Since r  D when t  0 , this constant is just the initial

32
CLASSICAL MECHANICS

displacement of the particle. It may be zero or not depending on the


reference point.

Example 2.1: The velocity of a particle moving with constant velocity


has a value v  (3, 1,  7) when the particle is at position

r  (1,  5,  2) . What is the distance the particle travels in 5 s ?

Solution: We take the instance the particle is at r  (1,  5,  2) as

t  0 . This implies that D  (1,  5,  2) . At this point,

v  v0  (3, 1,  7) . Substituting t  5 s and D  (1,  5,  2) into Eq.


(2.10), we obtain the position of the particle 5 s later as
r  (16, 0,  37) . The distance travelled is

s r  D   r  D   (16  1) 2  (0  5) 2  (37  2) 2


 152  52  372  225  25  1,369  1,619  40.24

2.1 Motion of a Particle in One Dimension

2.1.1 The Free Particle


We start our study of dynamics by considering the case of a particle
moving in one dimension (one direction only). We assume that the
motion is along the x axis, so that the equation of motion is

mx  F ( x, t ) (2.11)

If there is no force acting on the particle, the equation of motion of the


particle becomes

mx  0 (2.12)

Dividing Eq. (2.12) by m results in the differential equation

x  0 (2.13)

The solution of this equation is the x component of Eq. (2.10):

33
34

x  v0t  x0 (2.14)

where v0 is the velocity at time t  0 with corresponding position x0 .

2.1.2 Particle Moving Under the Effect of a Constant Force


Next we consider the case of a constant force k acting on a particle. We
assume that the particle is moving in one dimension taken as the
x  direction. The equation of motion is then

mx  k (2.15)

Using

dv
x  (2.16)
dt

and following the procedure of Eq. (2.9), we are able to write this as

k
dv  dt (2.17)
m

whose solution is

v  at  A (2.18)

where we have used the fact that k / m  a , the acceleration. (Remember


that we assumed a constant for F  k ) The result then follows from
Newton’s second law (Eq. 2.1).

If the velocity is v0 at t  0 , we find that

A  v0 (2.19)

and Eq. (2.18) becomes

v  at  v0 (2.20)

Using

dx
v (2.21)
dt

34
CLASSICAL MECHANICS

we can recast Eq. (2.20) into the form

dx  at  v0 dt (2.22)

Integrating this gives us the solution which is

1 2
x at  v0t  B (2.23)
2

with B as a constant of integration which has the dimensions of


displacement.

If we suppose that the initial conditions of the particle is

x  x0 when t  0 (2.24)

This fixes B , and gives

B  x0 (2.25)

The position as a function of time is then

1 2
x at  v0t  x0 (2.26)
2

Using the initial conditions t  0, x0  0 , we obtain

1 2
x at  v0t (2.27)
2

which we recognize as one of the five equations of rectilinear motion.


When a  g as in the case of a body moving under gravity, then

1 2
y gt  v0t (2.28)
2

In general, Eqs. (2.26) and (2.27) are sometimes written as

1
s  ut  at 2 (2.29)
2

where u  v0 is the initial velocity.

35
36

Example 2.2: A particle of mass 2 kg moves under the force

F  25 N . When passing the origin, the particle has a velocity of


 5 m/s . Where was the particle 10 s earlier?

Solution: The instance the particle passes the origin is at t  0 s . We


require the position at t  10 s . From the given information, the

acceleration is a  15 / 2  7.5 m/s2 . Thus using Eq.(2.27), we find that

the required position x  325 m.

2.1.3 The Simple Harmonic Oscillator


Simple harmonic motion is the case involving a particle acted upon by a
restoring force that is proportional to its displacement from the
equilibrium position. Denoting the displacement by x , the resulting
equation of motion is

mx  kx (2.30)

if the oscillating particle has mass m and k is the spring constant.

By making the substitution

k
2  (2.31)
m

where  is called the angular frequency, the equation of motion


becomes

x   2 x (2.32)

By means of an example, we shall show that the solution to this equation


is

x  x0 sint    (2.33)

where  is called the phase angle. We shall analyze this further when
we carry out detailed study on simple harmonic oscillators

Eq. (2.33) can be expressed as

36
CLASSICAL MECHANICS

x  x0 sin t cos  x0 cost sin  (2.34)

Example 2.3: A harmonic oscillator is at the positive extreme of its


motion at time t  0 . Determine the expression, x (t ) , that describes the
motion of the particle at any subsequent later time t .

Solution: Let the extreme be x  A , where A is the amplitude. The


initial conditions are therefore x  A and x  0 when t  0 . Now

x  x0 cost    (2.35)

By expanding Eq. (2.35), we have

x  x0 cost cos  x0 sin t sin  (2.36)

Substituting the initial conditions in Eqs. (2.35) and (2.36), we obtain the
following system of equations

x0 sin   A (2.37)

x0 cos  0 (2.38)

If we divide Eq. (2.38) by  the system of equations become

x0 sin   A (2.39)

x0 cos  0 (2.40)

Squaring the new system of equations and adding them gives

 
x02 sin2   x02 cos2   x02 sin2   cos2   A2 (2.41)

so that

x02  A2  x0  A (2.42)

In order to deduce the phase angle  , we proceed as follows: Divide Eq.


(2.39) by Eq. (2.40) to get

A
tan    (2.43)
0
37
38

The smallest angle for which this is true is    / 2, which means that

 
x  A sin t    A cost (2.44)
 2

2.2 General Treatment of Motion in One Dimension

When the force is a function only, there is a systematic way of treating


motion in one-dimension. Such a force which does not depend on time is
called static or conservative force.

This means that Eq. (2.11) is modified to read

mx  F (x) (2.45)

In order to solve this equation we apply some mathematical procedures.


First we multiply both sides of this equation by x to obtain

mxx  F ( x) x (2.46)

This can be written as

dx dx
mx  F ( x) (2.47)
dt dt

or

mxdx  F ( x)dx (2.48)

Integrating once we obtain

1 2
mx   F ( x)dx  E (2.49)
2

where E is a constant of integration. Since the left-hand side is the


kinetic energy, the term on the right-hand side must also be an energy, as
is the constant of integration.

Rewriting this as

1 2
mx   F ( x)dx  E (2.50)
2

38
CLASSICAL MECHANICS

The second term on the left-hand side is called the potential (energy and
is denoted by V (x) and defined as

V ( x)   F ( x)dx (2.51)

and is interpreted as the work done by the force F (x ) in moving the


particle through a distance x.

Eq. (2.50) the reads

1 2
mx  V ( x)  E (2.52)
2

The equation is telling us that the sum of the two energies on the left hand
side is always constant. It is a statement of the conservation of energy.

We can solve for the velocity from Eq. (2.52) and obtain

x  
2
E  V ( x )  (2.53)
m

By taking the positive root,

dx

2
E  V ( x )  (2.54)
dt m

or

dx
dt  (2.55)
2
E  V ( x ) 
m

By integrating this equation we find that

dx
t  (2.56)
2
E  V ( x ) 
m

where  is a constant of integration which has the dimensions of time.


Once we find t  t (x) , we can invert the expression to find x  x(t ) .
We now give some examples to illustrate these concepts.

39
40

Example 2.4: Starting from the fact that a particle moving under a
constant force F ( x)  k obey the stated principles above, show that if it

starts from the origin with velocity v0 at time t  0 , then its motion can

be described using Eq. (2.27).

Solution: For this case F ( x)  k and the potential energy is

V ( x)   kdx  kx (2.57)

In consequence, Eq. (2.56) can be written as

dx
t  (2.58)
2
E  kx
m

Since

   bx   bx 2
1 2 1
2 dx  (2.59)
b

it follows that

1
m 2  2
t   ( E  kx)   (2.60)
k m 

so that

1
m 2  2
 ( E  kx)   t  (2.61)
k m 

Using the initial conditions x  0, v  v0 at t  0 , then from Eq. (2.5),


we see that

1 2
E mv0 (2.62)
2

Employing this in Eq. (2.61) and remembering that this condition arises
when t  0 , we obtain

mv0
  (2.63)
k

Hence

40
CLASSICAL MECHANICS

1
m 2  2
mv0
 ( E  kx)  t (2.64)
k m  k

By squaring both sides we obtain

2mE 2mx 2 2mv0t m 2v 2


 t   2 (2.65)
k2 k k k

We can rewrite this as

2mx 2 2mv0t m 2v 2 2mE


t   2  2 (2.66)
k k k k

Dividing each term in Eq. (2.66) by 2m and multiplying by k , we


obtain

k 2 mv 2 E
x t  v0t  
2m 2k k

Using Eq. (2.62) while solving for x , we find

k 2 mv 2 mv02
x t  v0t   (2.67)
2m 2k 2k

Now bearing in mind that at t  0, x  0 , we see that Eq. (2.67) reveal


that

mv 2 mv02
 (2.68)
2k 2k

Hence

1 2
x at  v0t (2.69)
2

where we have used k / m  a . This is the same as Eq.(2.27) is the


required solution for the problem.

41
42

Example 2.5: Using the same approach as in example 2.4 above. (a)
show that for a simple harmonic oscillator in which the force is

2E
F ( x)   kx , the position at any time is given by x  sin  (t  t0 ) .
k
If the amplitude is b , (b) show that the total energy is given by
1 2
E kb .
2

Solution: (a) We similarly use Eq. (2.58) to find x (t ) . The potential


energy is

1 2
V ( x)     kxdx  kx (2.70)
2

so that

dx
t  t0 (2.71)
2 1 2
 E  kx 
m 2 

Therefore

1
m 2
dx
t  t0   
k
 b 2
 x2 
(2.72)

where

2E
b (2.73)
k

This integral can be solved by the substitution

x  b sin (2.74)

With

dx  b cos and b 2  x 2  b 2 (1  sin2  )  b 2 cos2  (2.75)

we have

1 1
m 2
b cosd  m  2
t  t0   
k
 b cos   k   (2.78)

42
CLASSICAL MECHANICS

Solving for  , we obtain

1
k 2
    (t  t0 ) (2.79)
m

and

 k  12 
x  b sin   (t  t0 ) (2.80)
 m  

Recalling that

1
k 2
   (2.81)
m

and using Eq. (2.73), we can write Eq. (2.80) as

sin (t  t0 )
2E
x (2.82)
k

(b) We know that the total energy must be the sum of the kinetic
energy and the potential energy, that is

1
Etotal  KE  PE  KE  kx2 (2.83)
2

We need to work out the kinetic energy and also obtain the maximum
potential energy.

1 2
KE  mx (2.84)
2

Using Eq. (2.82), we have

x  b cos (t  t0 ) (2.85)

For simplicity let (t  t0 )   , then Eq. (2.83) becomes

1 2 2 1
Etotal  mb  cos2   kb2 sin 2  (2.86)
2 2

43
44

We now use Eq. (2.81) to eliminate m and  2 from the first term of this
expression. The result is

1 2 1
Etotal  kb cos2   kb2 sin 2  (2.87)
2 2

Factoring out the common terms in Eq. (2.87) and using the trigonometric
identity cos2   sin2   1 , where    in this case, we finally get

1 2
Etotal  kb (2.88)
2

2.2.1 Time and Velocity Dependent Forces


Two categories of forces that we have not discussed in our treatment of
one dimensional dynamics are the time and velocity dependent forces.
There are the purely time-dependent and purely velocity dependent
forces. Sometimes we say that they are explicitly dependent. We shall
only discuss these forces very briefly because for the time-dependent
forces, the solutions can be obtained in two successive integrations with
respect to time. However, the velocity-dependent forces constitute a very
large topic which we have no time for in such a compressed module.

2.2.2 Time-Dependent Forces


If the force is explicitly dependent on time t , the differential equation of
motion for constant mass is

dv
F (t )  m (2.89)
dt

which can directly integrated to give

 F (t )dt  mv(t )  mv
0
0 (2.90)

44
CLASSICAL MECHANICS

The integral on the left-hand side of Eq. (2.90) is called the IMPULSE
and it is equal to the change of momentum imparted to a body by a force
F (t ) acting over a certain interval of time.

The position of the body as a function of time is obtained by a second


integration, that is, we can write from Eq.(2.90)

t
dx F (t )
 v(t )  v0   dt (2.91)
dt 0
m

Integrating both sides of this equation, we get

t
 F (t ) 
t t
x  x0   v(t )dt  v0t     dt dt (2.92)
0 0 0
m 

Provided the form of F (t ) and the initial conditions are given, the
position as a function of time can easily be obtained.

We shall consider as an example the case where the force is constant. For
this case we have

t
F Ft
v(t )  v0   dt  v0  (2.93)
m0 m

and

t
F
m 0
x(t )  x0  v0t  dt (2.94)

or

Ft 2
x(t )  x0  v0t  (2.95)
2m

which is equivalent to the equation of uniformly accelerated motion with


F
a , giving
m

at 2
x(t )  x0  v0t  (2.96)
2

45
46

For x0  0

at 2
x(t )  v0t  (2.97)
2

The other time-dependent forces that we are not going to discuss here are
the step forces. However, just to understand what these step forces are,
we provide a brief discussion.

Suppose that a body of mass m is subjected to a constant force F1 acting

for a time interval t1 and the force suddenly changes to a different

constant value of say F2 , then F2 is a step force. Another type of force


that is time-dependent is called “ the jerk”. It is a uniformly increasing
force.. Such a force can be represented by F (t )  ct , where c is a
constant. It is obvious that this force increases with time.

2.2.3 Velocity-Dependent Forces


Many situations in everyday occurrences exist in addition to constant
applied forces where the forces are functions of velocity. For example,
when a body is in a gravitational field, in addition to the gravitational
force, there exists a force of air resistance on the falling or rising body.
This resisting force is some complicated function of velocity. The same
is true for objects moving through fluids (gases and liquids). Such
opposing forces to the motion of objects through fluids are called
VISCOUS FORCES or VISCOUS RESISTANCE. In cases where they
exist, Newton’s second law may be applied in the following form:

dv
F (v )  m (2.98)
dt

or

dv dx dv
F (v )  m  mv (2.99)
dx dt dx

46
CLASSICAL MECHANICS

Knowing the form of the force F (v ) , either of the two equations Eq.
(2.98) or (2.99) may be solved to analyze the motion, that is, to calculate
x as a function of time t .

2.3 Summary
In this unit we have learned how to treat dynamical problems in one
dimension. The methods that have been presented are instructive and
therefore need to be generalized to the case of two and three dimensions.
This is the task in the next unit. From a formal point of view, the
equations that have been given can be used to obtain exact solutions of
the problem; however, there are many practical difficulties in
implementing them. For example, the integrals that need to be performed
may be intractable, but there are so-called numerical methods for treating
such cases. As far as we are concerned in this unit we have been able to
solve the one dimensional problem under position dependent-force
completely.

2.4 Exercises
2.0 A pendulum of length l is pulled to the side and by an angle  0

to the vertical and allowed to oscillate. Ignoring friction,

(i) Obtain the equation of motion of the pendulum.

(ii) Show that if the angle of swing is small, the pendulum performs
simple harmonic motion.

(iii) Obtain the equation if the displacement is maximum at t  0 .

2.1 The position of a particle moving along the x axis depends on


the time t according to the equation

47
48

v0
x (1  e  kt )
k

where v0 and k are constants.

(i) Obtain the velocity and the force acting on the particle

(ii) Calculate the maximum distance covered by the particle.

2.3 Calculate the potentials of the following forces

(i) F ( x)  ax  b (ii) F ( x)  F0e x / a (iii) F ( x)  x / a

(iv) F ( x)  F0 sin kx

2.4 The force acting on a particle of mass 1.5 kg is

F ( x)  7t 5  3t 2  2

in Newtons. Calculate

(i) The momentum of the particle.

(ii) The acceleration of the particle.

(iii) The position of the particle as a function of time.

2.5 A particle moves in a straight line under the action of a force


whose potential energy is given by

V ( x)  ax2 (b  x)

(i) Find the force acting on the particle.

(ii) Obtain the position of the particle as a function of time.

2.6 A body falls from a height of 1,000 km towards the surface of


the earth. Neglecting air resistance, write down the equation of motion of
the body and hence find the velocity with which it hits the earth.

2.7 A particle of mass m is released from rest a distance b from a


fixed origin of force that attracts the particle according to the inverse-
square law

F ( x)  kx2

48
CLASSICAL MECHANICS

Show that the time required to reach the origin is

1
 mb3  2
T    
 8k 

2.8 Find the velocity as a function of displacement x for a particle of

mass m which starts from rest at x  0 , subject to the following forces:

 cx
(i) F ( x)  F0  cx (ii) F ( x )  F0 e (iii) F ( x)  F0 coscx

2.9 A particle of mass m moves along a frictionless horizontal plane


with speed v ( x )   / x , where x is its distance from the origin and 

is a positive constant. Find the force F (x ) which acts on the particle.

2.10 A particle moving in a straight line along the positive x axis is

subject to a force F ( x)  b / x 3 , where b is a positive constant.

(i) Determine the dimensions of b .

(ii) Find an expression for the total energy of the particle.

(iii) Find the position of the particle as a function of time.

49
50

Chapter 3
Unit 3 Particle Dynamics in Two and Three
Dimensions

You should be able to cover this unit in 20 hrs. This time period does not
include the time for solving problems in the exercises. You may solve the
exercises at your own convenient pace.

OBJECTIVES

(1) To use the knowledge acquired in unit 2 to tackle problems in two and three dimensions

(2) To solve specific problems concerning central force.

After studying this unit, the learner should be able to:

(1) Solve two and three dimensional problems involving position dependent forces.

(2) Identify the salient properties of central forces in different systems.

(3) Solve problems involving central force motion.

(4) Solve specific standard conservative force problems in two and three dimensions.

50
CLASSICAL MECHANICS

(5) Identify and enumerate the properties of central force motion.

(6) Define work.

(7) Solve problems involving conservation of energy.

(8) Use the Del. Operator to solve conservative force problems.

(9) Use the Del. Operator to generate the components of angular momentum in three dimensions.

3.0 Introduction
In the previous unit, we studied the dynamics of a particle in one
dimension. Many important dynamical systems fall in this category. But
nature is three-dimensional and we naturally have to extend our
discussion to the case of particles moving in three dimensions. There are
also important situations where two-dimensional motion exists. In this
unit we generalize the theory we developed earlier to the case of two and
three-dimensional motion.

3.1 General Theory


In three-dimensional motion, the force is in general given by F ( r , t ) .
The equation of motion is then

mr  F (r , t ) (3.1)

We shall confine ourselves to static or time-independent forces. Thus


F  F (r ) , this makes the treatment of the problem somewhat simpler.
The vector equation (3.1) can be written in Cartesian-component form as

mx  Fx
my  Fy (3.2)
mz  Fz

In some cases the form of the force may warrant that the motion be
treated using a different coordinate system. If spherical polar coordinates
are used, then the component form of the equation of motion is

51
52

m(r  r 2  r sin 2 2 )  Fr


m(r  2r  r sin cos2 )  F
 (3.3)
m(r sin  2r sin  2r cos )  F

If the force is directed along the position vector, it is called a central


force. For this type of force, the equation of motion takes a simple form
because the force does not depend on the radial coordinate  and  and
so can be expressed as

F  f (r )rˆ (3.4)

In that case, although the problem is normally three-dimensional, the


motion takes place on a plane (may be xy, xz, or yz ). This will
depend on the plane of choice. The components of the equation of
motion are

m(r  r 2 )  f (r )
(3.5)
m(r  2r)  0

For the general case however, the procedure to be followed in tackling


Eq. (3.1) is analogous to the one that was adopted for the one-
dimensional motion case. We take the dot product of Eq. (3.1) with r to
obtain

mr  r  F  v (3.6)

This can be written as

mv  v  F  v (3.7)

We know that

v2  v  v (3.8)

and

d (v 2 ) d dv dv dv
 (v  v )  v    v  2v  (3.9)
dt dt dt dt dt

It follows that by dividing the last expression and first expression by 2 we


obtain

52
CLASSICAL MECHANICS

1 d (v 2 ) dv
v (3.10)
2 dt dt

We can then substitute these expressions in Eq. (3.7) to obtain

1 d (v 2 ) dr
m F (3.11)
2 dt dt

which yields

1
m d ( v 2 )  F  dr (3.12)
2

When integrated, this gives us

1
m v 2   F  dr  E (3.13)
2

where E is a constant of integration and is identified as the total energy.


This equation is evidently the counterpart of Eq. (2.50) and is a statement
of the conservation of energy and takes the form

K  V (r )  E (3.14)

for the three-dimensional motion. Here K represents the kinetic energy


while the potential energy

V (r )    F  r (3.15)

Many important systems can be treated by means of these results.

3.2 Particle in the Field of Gravity


Problems of particle motion in the field of gravity are quite common as
occurring near the surface of the earth so that the force acting on the
particle is taken to be constant. We shall now take into account the
variation of this force with distance from the centre of the earth.
According to Newton’s law of universal gravitation, the force with which
the earth acts on a particle of mass m in its vicinity is given by

53
54

GMm
Fr   (3.16)
r2

where M is the mass of the earth, G is the universal gravitational


constant and r is the distance of the particle measured from the centre of
the earth to the centre of the particle. We have taken in this case the earth
to be a uniform sphere so that its gravitational attraction is equivalent to
that of a point mass M located at its centre. Since we are dealing with a
spherical system (3-Dimensions), the equation of motion has three
components. For the present problem, we shall ignore the components

corresponding to the directions ˆ and ˆ . We shall focus on the radial


component and write

GMm
mr   (3.17)
r2

If the particle is at a distance x above the earth’s surface then the


distance of separation is

r  Rx (3.18)

It is clear from the above equation that r  x , since the radius of the
earth R is constant. Eq. (3.17) may then be written as

GMm
mx    Fx (3.19)
( R  x) 2

The potential energy is therefore

GMm
V ( x)     dx
( R  x) 2
(3.20)
GMm

Rx

And the energy conservation equation is

1 2 GMm
E mv  (3.21)
2 Rx

Now, if the body is projected upwards with a velocity v0 at the surface of

the earth, it follows from Eq. (3.21) that

54
CLASSICAL MECHANICS

1 2 GMm 1 2 GMm
mv0   mv  (3.22)
2 R 2 Rx

From this we have

2GM 2GM
v02  v 2   (3.23)
Rx R

The maximum height H reached by the body will correspond to v  0


and is therefore such that

2GM 2GM
v02   (3.24)
RH R

If we want to project a particle fast enough to make sure that it escapes


the gravitational pull of the earth, we must ensure that H   . This
gives the speed of projection as

2GM
v0  (3.25)
R

From

GMm
 mg (3.26)
R2

the equation which holds at the surface of the earth, the acceleration due
to gravity g can be written as

GM
g (3.27)
R2

so that Eq. (3.25) can be expressed as

v0  2 gR (3.28)

This quantity v0 is called the escape velocity and is the minimum

velocity which in the absence of air resistance or air friction, we must


project a particle from the surface of the earth to ensure that it escapes the
earth’s gravity. Since g  9.8 m/s2 and R  6.4  106 m ,

55
56

v0  11 km/s . Similar calculations can be carried out for other planets


when their gravitational constants are known.

3.3 Force and Work


We can now carry out a more detailed study on the potential energy Eq.
(3.15). We start by considering a force F which acts on a particle and
moves it through the displacement s . The work done cannot be
calculated by taking the scalar product of the force and the displacement
because the force is position dependent and therefore not constant.
Additionally, to reach the end of the displacement there are many paths
that can be followed. However, if the displacement dr is small enough,
the force is essentially constant over the infinitesimal distance dr and
the work done on the particle is

dW  F  dr (3.29)

The work when the particle is moved from ri to r f is

rf

W   F  dr (3.30)
ri

The work done in moving the particle from r to r f is by definition the

potential energy or potential or potential energy function. This is given


by

rf r
V (r )   F  dr    F  dr    F  dr  C (3.31)
r rf

where C is a constant of integration.

3.4 Force and Work


Statement of the work principle: The work done on a particle by a force
increases the kinetic energy of the particle by the same amount. To
demonstrate this, we start by looking again at Eq. (3.11)

56
CLASSICAL MECHANICS

1 d (v 2 ) dr
m F (3.32)
2 dt dt

This can be rewritten as

d 1 2 dr
 mv   F  (3.33)
dt  2  dt

By defining

1 2
K mv (3.34)
2

this shows that

dK  F  dr (3.35)

Therefore

 dK   F  dr (3.36)

Since the right-hand side of this equation is the work done on the particle
by the force, we see that this equation states that the work done on a
particle by a force increases the kinetic energy of the particle by the same
amount.

Example 3.1: A body of mass 3 kg and velocity 2.3 m/s is acted upon

by the force F ( x)  5 x  2 x 2 from point x  0 to x  2 . What is the


final speed of the body?

Solution: The change in the kinetic energy of the body is

2
 5 x 2 2 x3 
 
2
K   5 x  2 x dx  
2
  4.667J
3  0
(3.37)
0  2

57
58

1 2
The original kinetic energy was K i  mv i  7.935J . This shows that
2
if we add the difference to this value we should obtain the final energy.
This works out to be K f  12.602 J .

Using Eq. (3.34) the corresponding final velocity is

2K f
vf   2.90 m/s (3.38)
m

3.5 Force and Work (The Del Operator)


We have already studied the different types of forces and seen how they
arise in different situations. We shall go into more details on
conservative forces and develop a more general approach in dealing with

them. The integral  F  dr is called a line or path integral because it is

evaluated along the path of the particle. The exact value of the integral
depends on the path taken by the particle between the initial point ri and

the final point r f . However, for certain forces, this integral is path

independent. This is true for all position-dependent forces F (r ) . For


such forces, it must be that the integral is given by the difference in the
values of some function at the end points. The function is denoted by
U (r ) , so that

rf

 F  dr  U (r )  U (r )
ri
f i (3.39)

This equation means that the integrand is an exact differential meaning


that

 F  dr   dU  U (r ) (3.40)

This of course can be written in Cartesian components as

 F  dr   F dx  F dy  F dz
x y z (3.41)

58
CLASSICAL MECHANICS

In order to proceed from here, we digress a bit and give a brief


mathematical discussion on functions and their dependence on
independent variables.

If an arbitrary function G depends on the independent variables


x, y and z , then the change in G when the variables change from
x to x  dx, y to y  dy and z to z  dz is

G G G
dG  dx  dy  dz (3.42)
x y z

and dG is an exact differential such that

 dG  G (3.43)

This shows us that if F  dr is an exact differential, there is a scalar


function such that the components of F are just the partial derivatives of
this function with respect to the coordinates. If we let this function be
U (r ) , then we have

U U U
Fx  , Fy  , Fz  (3.44)
x y z

so that

U U U
F  dr  dx  dy  dz (3.45)
x y z

This ensures that F  dr is an exact differential and the integral of this


quantity between two points is given by the difference in the values of U
at those points. It is customary to introduce a negative sign so that the
scalar function is

V (r )  U (r ) (3.46)

Then

dV (r )  dU (r )   F  dr (3.47)

and

59
60

V (r )   F  dr (3.48)

V (r ) is called the potential energy.

By employing Eq. (3.45), F  dr  dV (r ) can be expressed in


rectangular coordinates as

V V V
Fx dx  Fy dy  Fz dz   dx  dy  dz (3.49)
x y z

This implies that

V V V
Fx   , Fy   , and Fz   (3.50)
x y z

We can then express the force F as

V ˆ V ˆ V ˆ
F  i j k (3.51)
x y z

By defining

 ˆ  ˆ  ˆ
 i j k (3.52)
x y z

we see that Eq. (3.51) can be written in a compact form as

V ˆ V ˆ V ˆ
F  V   i j k (3.53)
x y z

This can be compared with the one dimensional form

dV ( x)
Fx   (3.54)
dx

with similar expressions for the y and z components. The differential


operator defined in Eq. (3.51) is a vector operator called the del operator.
The expression  V is called the gradient of V and is sometimes written
grad V .

The negative sign in Eq. (3.53) implies that the particle is urged to move
in the direction of increasing potential energy rather than in the opposite
direction

60
CLASSICAL MECHANICS

Suppose the potential energy function exists so that Eq. (3.50) holds, then
we can take the derivative of Fx with respect to y and the derivative of

Fy with respect to x to obtain

Fx  2V Fy  2V
 ,  (3.55)
y yx x xy

Since the order of differentiation can be reversed on the right-hand side


of each expression in Eq. (3.55), we see that the expressions are equal
and we can write

Fx Fy
 (3.56)
y x

Performing similar operations on Fz we get

Fx Fz Fy Fy


 and  (3.57)
z z z y

Eqs. (3.56) and (3.57) are the necessary conditions on


Fx , Fy and Fz for the potential energy function to exist and they
express that condition that

F  dr  Fx dx  Fy dy  Fz dz (3.58)

We can now introduce the cross product of the del operator on F given
as

 F F   F F   F F 
  F   z  y iˆ   z  x  ˆj   y  x kˆ (3.59)
 y z   x z   x y 

This is sometimes written with a change of sign in the second term and
the terms in the bracket switched giving

 F F   F F   F F 
  F   z  y iˆ   x  z  ˆj   y  x kˆ (3.60)
 y z   z x   x y 

Eq. (3.59) or (3.60) is what we call the CURL of F . If we substitute Eq.


(3.56) and (3.57) into Eq. (3.60) we see that

61
62

F 0 (3.61)

since each term vanishes separately. This is the condition for a force to
be conservative.

We see from Eq. (3.60) that we can write

iˆ ˆj kˆ
  
F  (3.62)
x y z
Fx Fy Fz

3.6 Angular Momentum


There are other dynamical quantities of interest that arise for a particle
moving in three dimensions. One of these quantities is the angular
momentum L . The quantity is conserved whenever the moment of the
force about the origin is zero. To show this we consider the equation of
motion given by Eq. (2.3)

mr  F (3.63)

Taking the cross product of this equation with the position vector gives

r  mr  r  F (3.64)

The right-hand side is the moment of the force F about the origin. We
want to analyze the left-hand side.

We can rewrite Eq. (3.64) as

dv dp
r F  r m r (3.64)
dt dt

The right-hand side can be simplified further by noting that

d dp dr dp
(r  p )  r   pr v p (3.65)
dt dt dt dt

and since v  p  v  mv  m(v  v )  0 because (v  v )  0 , Eq


(3.64) can be written as

62
CLASSICAL MECHANICS

d
rF  (r  p ) (3.66)
dt

If we define L  (r  p )  r  mr , then Eq. (3.66) becomes

dL d  dr  dr
 (r  mr )     mr  r  m (3.67)
dt dt  dt  dt

This is similar to Eq. (3.65) and the first term vanishes and we conclude
that

dL d dr
 (r  mr )  r  m  r  F  M (3.68)
dt dt dt

Thus the moment of the force F , about the origin is equal to the time
derivative of the angular momentum. If the moment is zero, then

dL
0 (3.69)
dt

This implies that

L  constant (3.70)

We thus arrive at the law of conservation of angular momentum for a


particle. The law states that: If the moment of the net force acting on a
particle is zero, the angular momentum of the particle about the origin
is constant. We have seen that the angular momentum is just the cross
product of the position vector and the linear momentum. This means that

L rp (3.71)

and its direction is given by the right-hand rule. We move our right hand
as if we are manipulating a screw driver. If the rotary motion is from the
vector r to the vector p then L points in the direction in which the
screw would go

In view of Eq. (3.59) and (3.62) we can express the components of L in


a compact form as

63
64

iˆ ˆj kˆ
L rp x y z (3.72)
px py pz

Therefore

Lx  ypz  zpy
Ly  zpx  xpz (3.73)
Lz  xpy  ypx

3.7 Central Force Motion


Now that we understand what angular momentum is, we shall treat an
important category of three-dimensional motion governed by a central
force. A central force is a static (conservative) force whose magnitude
depends only on the radial coordinates and whose direction is along the
position vector, so that it points either from the origin of the particle or
from the particle to the origin. For a central force therefore, we have Eq.
(3.4).

F  f (r )rˆ (3.74)

We have seen that the angular momentum L of a particle is related to the


moment M on the force acting on the particle by Eq. (3.68)

dL
rF  M (3.75)
dt

and for a central force, it is

M  rrˆ  f (r )rˆ  rf (r )rˆ  rˆ  0 (3.76)

So a central force has no torque about the origin and therefore

dL
0 (3.77)
dt

so that

64
CLASSICAL MECHANICS

L  constant (3.78)

We see that the angular momentum of a particle moving under the effect
of a central force is a constant. This result is important and implies that
far from being three-dimensional, central force motion takes place in a
plane. The proof of this is as follows.

Since the angular momentum is defined as

L rp (3.79)

it is a vector at right angles to the plane containing the position vector and
the linear momentum. But since L is constant, it is in particular constant
in both magnitude and direction. Hence the plane containing position
vector r and the velocity vector v is fixed in orientation because if this
orientation is changing, then L would also change. At all times the
particle has a position vector lying in this fixed plane. We conclude that
motion under a central force always takes place in a plane of fixed
orientation.

Central force motion owes a great deal of its importance to the fact that
the gravitational force is central. Thus the motion of the earth about the
sun is characterized by a fixed angular momentum and is planer. The
motion about the moon is the same. In fact any celestial body orbiting
another moves in a plane. Another important central force is the
electrostatic force between charged particles. These two forces, the
gravitational and electrostatic, are among the most important in nature.

For a central force, the potential energy is

V (r )   f (r )rˆ  dr   f (r )dr (3.80)

where we have used the definition and property of the dot product of two
vectors as given by Eq. (1.1). Using the expressions for the components
of the acceleration in spherical polar coordinates, we find from Newton’s
second law

mr  F (3.81)

65
66

that components of the equation of motion for motion in a central force


field are

m(r  r 2  r sin2 2 )  Fr  f (r )


m(r  2r  r sin cos2 )  F  0  (3.82)
m(r sin  2r sin  2r cos )  F  0

which follow from Eq. (1.71) and (3.3). If we chose polar coordinates
r ,  in the plane of motion, only the first and second equations in Eq.
(3.82) hold and can be rewritten as

m(r  r 2 )  f (r )
(3.83)
m(r  2r)  0

since the motion does not depend on  which is therefore suppressed.

Now from

r  rrˆ( ) (3.84)

we have

v
d
rrˆ( )  r  rrˆ  rˆ (3.85)
dt

as given by Eq. (1.23). The acceleration follows from Eq. (3.85) and is

  
a  r  r rˆ  r  2r ˆ  (3.86)

as given by Eq. (1.29). Thus the two components of the acceleration a


are the radial acceleration ar and the angular acceleration a given by

Eq. (1.30) and (1.31)

ar  r  r 2 (3.87)

a  r  2r (3.88)

Since the force is central, the F  0 and F  0 . Hence, Eq. (3.83).

Let us rewrite the second equation in Eq. (3.83) as follows

66
CLASSICAL MECHANICS

d d dL
(mr 2)  (mrr)  0 (3.89)
dt dt dt

Integrating this we obtain

mr 2  L  0 (3.90)

The constant L is to be evaluated from the initial conditions.

Since the force is conservative we must have

T V  E (3.91)

Using Eq. (3.85) we have

T
1 2 1
2 2
  1
 
mv  m rrˆ  rˆ  rrˆ  rˆ  m r 2  r 2 2
2
 (3.92)

or

T
1
2
  1
m r 2  m r 2 2
2
  (3.93)

which we can see as composed of translational and rotational energy.

Eq. (3.91) can then be written as

1 2 1 2 2
T V  mr  mr   V (r )  E (3.94)
2 2

where V (r ) is the potential energy or potential energy function given by


Eq. (3.48)

rf

V (r )  V (r )    F  dr (3.95)
ri

and E the constant energy to be evaluated from the initial conditions

Now from

mr 2 2  L (3.96)

we can rewrite Eq. (3.94) as

67
68

1 2 L2
T V  mr   V (r )  E (3.97)
2 2mr 2

We can solve this equation for r to get

2 L2 
v  r   E  V (r )   (3.98)
m 2mr 2 

Therefore

r
dr 2
 2 L  2

m
t (3.99)
 E  V (r )  
r0

m 2mr 2 

The integral can be evaluated and the resulting equation solved for r (t ) .

For  (t ) we use mr 2 2  L to get

t
L
  0   (3.100)
0
mr 2

We have thus so far obtained the solutions of Eq. (3.83) in terms of the
four constants L, E , r0 and 0 which can be evaluated when the

initial position and the velocity in the plane are known. Furthermore the
potential energy function can have many forms such as the Yukawa
potential in nuclear particles the potential function is taken to be of the
form

ke ar
V (r )  (3.101)
r

where k and a are constants and k  0

For two masses M and m separated by a distance r in a gravitational


field, the potential (potential energy) or potential energy function is

GMm GMm
V (r )     2
dr   C (3.102)
r r

with C the constant of integration and has the same units as V (r ) .

Physically C represents the initial potential energy.

68
CLASSICAL MECHANICS

3.8 Summary
In this unit, we have generalized to the case of two and three dimensions
the methods seen in unit 2 for handling dynamical problems. From the
knowledge gained, it is now possible to treat such important problems
such as the revolution of the planets about the sun and motion of charged
particles in an electric or magnetic field force. The theory we have
developed so far can be use very conveniently to point particles and the
dynamics of aggregate of particles and rigid bodies will be presented later
in the module.

3.9 Exercises
3.0 Show that the velocity and acceleration of a particle moving
in a circle are perpendicular if and only if the particle is moving
with a constant velocity.

3.1 A particle of mass m moves according to the equations

x  x0  at 2 , y  bt 2 , z  ct

where x0 , a, b, c are constants.

(a) Find the momentum L at any time t .

(b) Find the force F and from it the torque N acting on the
particle
(c) Verify that the angular momentum theorem

dL / dt  r  F  N is satisfied.

69
70

3.2 Consider the one-dimensional potential

U ( x) 

 Wd 2 x 2  d 2 
x 4  8d 4

Sketch the potential and discuss the motion at various


values of x. Is the motion bounded or unbounded? Where
are the equilibrium values? Are they stable or unstable?
Find the turning points for E  W / 8 . The value of W is a
positive constant.

3.3 A particle is under the influence of a force

kx3
F  kx 
a2

where k and a are positive constants.


(a) Obtain an expression for the velocity
(b) Determine U (x) and discuss the motion,

3.4 A particle is projected vertically upwards with a speed v


from a height h . Show that the particle hits the ground

with speed v  v02  2 gh

3.5 A particle of mass m is subject to a force

 x3 
F  k   x  2 
 a 

(i) What are the SI units for a and k ?


(ii) Obtain an expression for the potential energy V (x)
assuming that V (0)  0 .

70
CLASSICAL MECHANICS

3.6 A locomotive of mass m moving with a velocity v in a


straight line is pulled with a force P against a resistance
R . Show that the distance travelled by the locomotive in
accelerating from a velocity v1 to v 2 is given by

v2
vdv
d  m .
v1
PR

When the speed of the locomotive is v 3 , its engine is turned


off and it comes to rest under the action of a resistance
R  a  bv .

Find an expression for the distance travelled.

3.7 If a block is projected with an initial velocity v 0 on a


smooth horizontal surface and that there is no air resistance
such that the block moves under a force F (v)  c1v .
Write down the differential equation describing the motion
if the block has a mass m . Find an expression for the
position at any later time t .

71
72

Chapter 4
Unit 4 Simple Harmonic Motion

You should be able to cover this unit in 20 hrs. This time period does not
include the time for solving problems in the exercises. You may solve the
exercises at your own convenient pace.

OBJECTIVES

(1) To teach the learner how to treat both free and damped harmonic oscillators.

(2) To teach the learner how to obtain the frequency of vibration of disturbed systems.

After studying this unit, the learner should be able to:

(1) Identify a free and damped simple harmonic oscillating system.

(2) Write down the equation of motion for the identified system in (1) above.

(3) Solve the problem of the system whose equation has been written in (2) above.

(4) Obtain solutions of free, underdamped (weakly damped), overdamped (strongly damped) and
critically damped systems.

72
CLASSICAL MECHANICS

(5) Calculate amplitudes, frequencies, positions, velocities and accelerations of different oscillating
systems.

4.0 Introduction
All around us there are objects which vibrate or oscillate. Swings in
playgrounds, musical instruments, water waves, atoms and molecules
moving in matter are some examples of vibrations and oscillations. A
force acting on a particle displacing it from its equilibrium in the
direction opposed to its displacement will always result in to and fro
motion. If the magnitude of this force is f (x ) , where x is the
displacement then for vibratory motion to occur, we must have

ma   f (x) (4.1)

where m is the mass and a is the acceleration. Since the force  f (x)
always pulls the mass m back towards the position of equilibrium, it is
called a restoring force.

Most important vibrations correspond to the case where f (x ) is directly


proportional to the displacement, so that we can write

f ( x)  kx (4.2)

where k is a constant called the force constant. In some literature it is


also called the spring constant because a vibrating body behaves as if is
connected to a spring. In view of E. (4.2), the equation of motion is

mx  kx (4.3)

A body whose motion is governed by this equation is called a simple


harmonic oscillator. We see that the conditions for a body to undergo
simple harmonic motion (often abbreviated SHM) are:

(i) There must be a restoring force

(ii) The magnitude of this restoring force must be proportional to the


displacement

73
74

There are many examples of simple harmonic oscillators such as a mass


connected to a spring and hung vertically so that the mass is free to move
up and down when displaced, a pendulum displaced slightly from its
equilibrium position and a floating body pushed further into the liquid
and released. Simple harmonic motion is so widespread that any system
whatsoever will perform simple harmonic motion if displacement from
the equilibrium position is small enough. Such is the case when a guitar
string is plucked or when a drumhead is beaten. At microscopic level,
atoms and molecules perform simple harmonic motion when excited by
thermal energy from the environment.

4.1 Displacement in Simple Harmonic Motion


We now proceed to solve Eq.(4.3) by introducing the angular frequency
through the equation

k
 (4.4)
m

the equation of motion can be written as

x   2 x (4.5)

Simple harmonic motion can in fact be thought of as the projection of the


position vector of a particle which rotates with constant angular velocity
 on the circumference of a circle. If the frequency of the motion is f ,
then the particle makes f revolutions per unit time and in that time the

angle rotated is 2f . We can also say that the period of oscillation is
1 / f . In this period, the angle rotated is 2 . If the angular velocity is
 , we see that time taken for one revolution is 2 /  . The connection
between  and f is therefore

2 1
 T (4.6)
 f

This is normally written as


f  (4.7)
2
74
CLASSICAL MECHANICS

An inspection of Eq. (4.5) shows that the solution that we seek is a


function with the property that when it is differentiated twice, it remains
unchanged but acquires a multiplicative constant. Of all the known
functions, the only ones with this property are the sine, the cosine and the
exponential functions. Since sine or cosine functions can be expressed in
exponential form we choose to express the required solution in
exponential form because the ease with which exponential functions can
be manipulated in terms of differentiation. We let the solution be

x  e t (4.8)

where  is to be determined.

Differentiating this once we get

x  et (4.9)

and a second differentiation gives

x  2 e t (4.10)

Substituting these expressions in Eq. (4.5) we obtain

2et   2et (4.11)

Cancelling the non-zero factor et we have

2   2 (4.12)

which is satisfied by   i and   i (where i 2  1 ) and so

eit and eit (4.13)

are both solutions of the SHM differential equation. Since the equation is
linear the general solution will be a linear combination of these two
solutions with arbitrary constant coefficients, that is

x  Ae  it  Be  it (4.14)

The two constants can be determined from the so-called initial conditions

x(t  t0 )  x0 (4.15)

75
76

and

x (t  t0 )  v0 (4.16)

These constants have been introduced because the solution of any


second-degree differential equation demands this. The reason is that
when the differential equation was obtained by differentiating the
solution twice, two constants must have been lost and they have to be
recovered. The initial conditions evidently give us the value of the
solution and of its first time derivative at a certain instant of time. Since
there are two unknowns and two equations from the initial conditions,
these constants can be determined from the initial conditions.

The information required to fix A and B can also be provided in the


form of the so-called boundary conditions

x(t  t1 )  x1 (4.17)

x(t  t2 )  x2 (4.18)

or in the form of

x (t  t1 )  x1  v1 (4.19)

x(t  t2 )  x2  v2 (4.20)

By means of Euler’s formula

e  i  cos  i sin (4.21)

we can expand Eq. (4.14) as

x  A cost  iA sin t  B cost  iB sin t


 ( A  B) cost  i( A  B) sin t
(4.22)
 x0 sin  cost  x0 cos sin t
 x0 sin(t   )

where

x0 sin   ( A  B) (4.23)

and

76
CLASSICAL MECHANICS

x0 cos  i( A  B) (4.24)

We can similarly convert expression (4.12) to the cosine form

x  x0 cos(t   ) (4.25)

x0 is the amplitude while  and   are called phase angles- ( some


literature use  and   ). When the phase angle is zero, the general form
of x (t ) is illustrated in Fig. 4.1

x(t )  A sin t

0 Time

Period T

Fig. 4.1: Position plotted against time in SHM

The frequency f is just the number of complete oscillations the


oscillator makes in unit time. What of  ?

The angular frequency is so named because of the intimate connection


between the circular motion and simple harmonic motion. We suppose
that a particle moves with a constant angular velocity  in the xy -plane

in a circle of radius x0 centered on the origin. If we shine light straight

down the y axis, the shadow of the particle will be seen on a horizontal

screen placed below the x axis at any distance not greater than x0 . It

can be shown that the shadow performs horizontal simple harmonic


motion along the x axis with the origin as the equilibrium position. The

77
78

period of this motion corresponds to the time it takes the particle to make
one complete revolution. The angular frequency is just the angle through
which the particle rotates in unit time. This is clearly 2f , the
expression for  (Eq. (4.6) or (4.7)).

4.2 Velocity and Acceleration in Simple Harmonic Motion


Now that we know the displacement of simple harmonic motion, we can
proceed to calculate the velocity and acceleration of such motion. The
velocity and acceleration in simple harmonic motion given by Eq. (4.22)

x  x0 sin(t   ) (4.26)

are given by

dx
 x  x0 cos(t   ) (4.27)
dt

and

d 2x
 x  x0 2 sin(t   ) (4.28)
dt 2

One can proceed and use Eq. (2.25) instead.

The maximum value of the velocity x0 is called the velocity amplitude

and the acceleration amplitude is given by x0 .


2

4.3 The Energy of a Harmonic Oscillator


The fact that the velocity is zero at maximum displacement in simple
harmonic motion and is a maximum at zero displacement illustrates the
important concept of an exchange between kinetic and potential energy.
In an ideal case, the total energy remains constant but this is never
realized in practice because of losses. If no energy is dissipated then all
the potential energy becomes kinetic energy and vice versa, so that the
value of (a) the total energy at any time (b) the maximum potential
energy and (c) the maximum kinetic energy will be equal; that is

78
CLASSICAL MECHANICS

Etotal  KE  PE  KEmax  PEmax (4.29)

From this equation, it is evident that

1 2
Etotal  E  mx  V ( x) (4.30)
2

For a spring

F  kx (4.31)

and so

1 2
V ( x)     kxdx  kx (4.32)
2

Hence

1 2 1 2
Etotal  E  mx  kx (4.33)
2 2

Since E is constant we have

dE d  1  1 d
1
  mxx  kxx  mxx  kxx  0
dt dt  2 2  2 dt
 mxx  mxx  kxx  kxx   0
1
(4.34)
2
 mxx  kxx   mxx  kxx  0
2
2

By canceling the non-zero common term x , we again obtain the equation


of motion

mx  kx  0 (4.35)

The maximum potential energy occurs at x   x0 and is therefore

1 2
PEmax  kx0 (4.36)
2

The maximum kinetic energy is

79
80

1  1

KEmax   mx 2   mx02 2 cos2 (t   ) max 
2  max 2 (4.37)
1
 mx02 2
2

when the cosine factor is unity.

But from Eq. (4.2) m 2  k so the maximum value of the potential


energy and kinetic energies are equal, showing that the energy exchange
is complete.

The total energy at any instant of time or value of x is

1 2 1 2
E mx  kx
2 2
1 2 2

 mx0  cos2 (t   )  sin 2 (t   )
2

(4.38)
1
 mx 02 2
2
1
 kx02
2

as we expected

Fig. 4.2 shows the distribution of energy versus displacement for simple
harmonic motion. Note that the potential energy curve

1 2 1 2 2 2
PE  kx  mx0  sin (t   ) (4.39)
2 2

is parabolic with respect to x and is symmetric about x  0 , so that the


energy is stored in the oscillator both when x is positive and when it is
negative, e.g. a spring stores energy whether compressed or extended, as
does a gas in compression or rarefaction. The kinetic energy curve

1 2 2
KE  mx0  cos2 (t   ) (4.40)
2

Is parabolic with respect to both x and x . The inversion of one curve


with respect to the other displays the  / 2 phase difference between the
displacement (related to the potential energy) and the velocity (related to
the kinetic energy).

80
CLASSICAL MECHANICS

For any value of the displacement x the sum of the ordinates of both
curves equals the total constant energy E .

Fig. 4.2: Parabolic representation of potential energy and kinetic energy


of simple harmonic motion versus displacement. Inversion of one curve
with respect to the other shows a 900 phase difference. At any
displacement value the sum of the ordinates of the curves equals the total
constant energy E.

From the foregoing theory, we now take a look at the specific example of
a simple pendulum.

81
82

Example 4.1: The simple pendulum

L cos  L

h

Fig. 4.3: The motion of a simple pendulum

For a simple pendulum, the potential energy is gravitational as illustrated


in Fig. 4.3. The pendulum is of length L and is at an angle  to the
vertical. Its vertical height from the lowest position is

h  L  L cos (4.41)

and its potential energy is

V  mgh  mgL(1  cos ) (4.42)

Therefore

1 2 1 2
Etotal  E  mx  kx (4.43)
2 2

Since  is very small

2
cos  1  (4.44)
2

and

sin   (4.45)

Also, since the sideways displacement of the pendulum is

x  L sin  L (4.46)

then

x  L (4.47)

Hence

82
CLASSICAL MECHANICS

1 2 2 1
E mL   mgL 2 (4.48)
2 2

Example 4.2: A harmonic oscillator passes through the equilibrium


position when t  0 , Use the solution

x  Ae  it  Be  it (4.49)

to show that the position as a function of time is

x(t )  R sin t (4.50)

where R is the amplitude

Solution: The boundary condition given is x  0 when t  0 . This


gives

A B  0 or B  A (4.51)

Thus,

x  Ae  it  Ae  it (4.52)

Making use of Eq. (4.21) we get


x(t )  A e  it  e  it 
 e  it  e  it 
 2iA  (4.53)
 2i 
 R sin t

where R  2iA

83
84

4.4 Simple Harmonic Motion in Nature


Whenever a slight displacement from equilibrium of a physical system
takes place, the resulting vibration is simple harmonic. We now proceed
to prove this fact.

Consider a particle in one dimensional-motion which is in stable


equilibrium before it is disturbed. Let the particle be in stable
equilibrium and held by a potential V (x) and be located at a minimum of
the potential. Such an arbitrary potential is shown in Fig. 4.3 (where the
potential is denoted by U (x ) rather than V (x) ). Let a minimum of such

a potential be at x0 . Such a location is a point of stable equilibrium. At

the minimum, the potential does not change with position and therefore
satisfies

dV
0 (4.54)
dx

and since the force is given by

dV
F ( x)   (4.55)
dx

it vanishes at x  x0 . This is precisely why the particle is in equilibrium

at that point.

U (x ) E

a x0 b c x

Fig. 4.4: Minimum of an arbitrary potential

84
CLASSICAL MECHANICS

When the particle is slightly displaced, the new position of the particle is
x and the value of the force at that point can be expressed in terms of its
value at x0 by means of the Taylor expansion. We have

 dF  1  d 2F 
F ( x)  F ( x0 )    ( x  x0 )   2  ( x  x0 ) 2
 dx  x0 2!  dx  x
0
(4.56)
1  d 3F 
  3  ( x  x0 )3  
3!  dx  x
0

If ( x  x0 )  1 , then only the first two terms are sufficient and we have

 dF 
F ( x)  F ( x0 )    ( x  x0 ) (4.57)
 dx  x0

But F ( x 0 )  0 and from Eq. (4.55)

dF d 2V
 2 (4.58)
dx dx

This means that the force at x can be written as

 d 2V 
F ( x)    2  ( x  x0 ) (4.59)
 dx  x0

When evaluated at x0 the second derivative of the potential gives a

constant which we denote by k :

 d 2V 
k 2 (4.60)
 dx  x0

This constant is positive because the second derivative of any function at


a minimum of the function is positive. The quantity gives the rate of
change of the slope at that point and we see that the slope is gradually
changing from negative to positive value at the minimum, this quantity is
necessarily positive. It is technically called the curvature of the function.
We can then write the equation for the force as

F ( x)  k ( x  x0 ) (4.61)

85
86

and the equation of motion is

mx  F ( x)  k ( x  x0 ) (4.62)

If we denote the displacement by   ( x  x0 ) , we find that   x so

that the equation of motion is

m  k (4.63)

This is just the defining equation for simple harmonic motion as already
presented by Eq. (4.3).

We have established that whenever any particle or body is slightly


displaced from its equilibrium position, it subsequently performs simple
harmonic motion with a force constant given by the second derivative of
the potential evaluated at the position of stable equilibrium. In nature,
slight displacements of bodies from equilibrium positions are numerous.
As a result simple harmonic motion occurs everywhere, but normally
with such small amplitudes that cannot be detected by the naked eye.
However, such vibrations tend to produce sound if they fall in the correct
frequency range. Sound is therefore one of the proofs of the prevalence
of simple harmonic motion.

Example 4.3: A particle of mass m moving along the positive x axis is


acted upon by a force whose potential is

c1 c2
V ( x)   (4.64)
x4 x2

where c1 and c2 are positive constants. Show that the period of small
oscillations about the position of stable equilibrium is

c1 m
T  2 (4.65)
c2 c2

Solution: We first find the point of stable equilibrium. From

86
CLASSICAL MECHANICS

dV 2c2 4c1
 3  5 0 (4.66)
dx x x

we find that the critical point is at

2c1
x (4.67)
c2

To determine which of these points correspond to stable equilibrium, we


compute the second derivative of the potential at each of these points.

Now

d 2V 20c1 6c2
 6  4 (4.68)
dx2 x x

When

2c1
x (4.69)
c2

is inserted into Eq. (4.68), the result is

d 2V c23
 0 (4.70)
dx2 c12

since the constants are both positive. Hence the point is a minimum.

The force constant for small oscillations about a point of stable


equilibrium x  x0 is given by Eq. (4.60)

 d 2V 
k 2 (4.71)
 dx  x  x0

In this case therefore,

c23
k 2 (4.72)
c1

and from Eq. (4.4) and (4.6) we find

87
88

k c23 c c2
  2
 2 (4.73)
m mc1 c1 m

and

2 c2 c2
T  2 (4.74)
 c1 m

4.5 Rotary Motion and Simple Harmonic Motion


Simple harmonic motion occurs in many mechanical systems as the
projection of rotary motion onto a line. We may understand this by
referring to Fig. 4.5. The figure shows a particle rotating in the xy plane

with constant angular velocity  in a circle of radius A centered on the


origin. The position vector of the particle has magnitude A but changing
indirection all the time.

Asint

0 x

Acost

Fig. 4.5: SHM from rotary motion

The angle   t is the angle which the position vector makes with the
positive x axis in the counter-clockwise direction. Hence the x and y
coordinates of the particle are

x  Acos  Acost (4.75)

88
CLASSICAL MECHANICS

y  A sin  A sin t (4.76)

If when t  t0 the position vector lies along the positive x axis, then

  t and at any time

x  A cos (t  t0 ) (4.77)

y  A sin  (t  t0 ) (4.78)

We see that the coordinates of the particle perform simple harmonic


motion. Simple harmonic motion may therefore be viewed as the
projection of rotary motion on the coordinate axis

Example 4.4: A vertical spring attached to the ceiling supports a mass


m under whose weight the mass stretches by an amount L1 .. If further
pulled down by a distance L2 and released at time t  0 , the mass
oscillates. Determine

the velocity of the mass as it passes through the equilibrium position and

the acceleration of the mass when it is at the top of the motion.

Solution: Let the stiffness (spring) constant be k and let the downward
direction be taken as positive. Then by Hooke’s law we have

kL1  mg (4.79)

and find

mg
k (4.80)
L1

Hence the angular frequency is

k g
  (4.81)
m L1

The solution is therefore a linear combination of the two solutions

y  A cost  B sin t (4.82)

89
90

and the velocity at any time is

dy
 y  A sin t  B cost (4.83)
dt

Employing the initial conditions

y  L2 , y  0, when t 0 (4.84)

and applying the velocity condition first gives us

0  0  B, B0 (4.85)

Hence

y  A cost , y  A sin t (4.86)

The position condition gives

L2  A (4.87)

so that

y  L2 cost (4.88)

Therefore the velocity is

y  L2 sint (4.89)

while the acceleration is

y   2 L2 cost   2 y (4.90)

As the mass passes through the equilibrium position, the time is at


t  T / 4 , where T is the period. Hence

 2 1  g
y  L2 sin t  L2 sin  T   L2   L2 (4.91)
 T 4  L1

At the top of the motion, the position is y   L2

therefore,

y   2 L2 cost   2 y   2 L2 (4.92)

90
CLASSICAL MECHANICS

Substituting the expression for  from Eq. (4.81), we get

gL2
y  (4.93)
L1

4.6 Damped Oscillations


Theoretically, a linear or simple harmonic oscillator once set into motion
will continue oscillating forever. Such oscillations are called free
oscillations. In practice, however, in any physical system there are
dissipative or damping forces, and the oscillating system will lose energy
with time. Thus the oscillating system is damped and eventually comes
to rest. Damping is not just a nuisance but is an effect that finds
important applications in many areas of technology. For example, the
shock absorbers in cars are meant to quickly subdue any oscillations of
the springs even as they perform their function of making the ride
smooth.

Damping forces are usually functions of velocity. Roughly speaking, the


air friction experience by a car is proportional to the square of the
velocity at which it is moving. We need to know the functional
dependence of velocity on any damping force acting on a physical system
if we have to treat it theoretically.

In a system which is damped, the differential equation for a linear


oscillator given in Eq. (4.3) must be modified to include the effect of
damping.

We consider a mass m tied to a spring as shown in Fig. 4.6, as a


prototype and restrict its motion to one dimension.

As the mass moves in a fluid, air or liquid, the frictional force is the
viscous force that produces the damping. As long as the speed of the

91
92

mass is small so as not to cause turbulence, the frictional force or


damping force Fd may be assumed to be directly proportional to the

velocity. That is

Fd  bv  bx (4.94)

where b must be a positive constant.

Fig. 4.6: Force acting on a prototype of a damped harmonic oscillator.

The net force Fnet due to forces acting on the mass m as shown in Fig.

4.6 is

Fnet  F  Fd  kx  bx (4.95)

Using Newton’s second law and substituting Fnet  mx in Eq. (4.95), we

get

mx  bx  kx  0 (4.96)

Eq. (4.96) is a second-order differential equation for the damped


harmonic oscillator of the mass spring system shown in Fig. 4.6.

There are a number of techniques of solving this equation. The one we


shall use is as follows:

We divide both sides of the equation by m and substitute

92
CLASSICAL MECHANICS

b
 (4.97)
2m

and

k
02  (4.98)
m

to obtain

x  2x  02 x  0 (4.99)

As with the free harmonic oscillator, let us try an exponential solution of


the form

x  e t (4.100)

where  is to be determined.

Differentiating this once we get

x  et (4.101)

and a second differentiation gives

x  2 e t (4.102)

Substituting these expressions in Eq. (4.99) we obtain


et 2  2  02  0  (4.103)

Since e t  0 , we must have

 2

 2  02  0 (4.104)

This auxiliary or sometimes called the characteristic equation has the


roots

1     2  02 (4.105)

and

1     2  02 (4.106)

93
94

The solution of Eq. (4.99) is with A1 and A2 as arbitrary constants

x(t )  A1e1t  A2e2 t (4.107)

or

x(t )  et  A1e 


  2  02 t   2  02 t
 A2e (4.108)
 

The following cases of this solution are of special interest and will be
discussed in some detail.

Case (I) Underdamped or 02   2 1 and 2 are


weakly damped imaginary roots
motion
(complex)
(oscillatory)

Case (II) Critically damped 02   2 1 and 2 are real


(not oscillatory) and are equal

Case (III) Overdamped 02   2 1 and 2 are real


(not oscillatory) roots

Case (I) Underdamped Oscillations, 02   2 : For this case it is

convenient to make a substitution:

k b2
1     
2
0
2
 (4.109)
m 4m 2

Since 02   2 , this makes

k b2
1     
2
0
2

m 4m 2

imaginary,

and we may write

94
CLASSICAL MECHANICS


x(t )  et A1e i1t  A2ei1t  (4.110)

which is a solution of an under-damped oscillator. Using Eq. (4.21), we


may write Eq. (4.110) as

x(t )  et i A1  A2 sin 1t   A1  A2 cos1t  (4.111)

Substituting i A1  A2   B and  A1  A2   C , we obtain an alternative


solution

x(t )  et B sin 1t  C cos1t  (4.112)

This may be written in a slightly different form by making the following


substitution in Eq. (4.112).

C
A  B 2  C 2 and tan    (4.113)
B

Thus we obtain

x(t )  Ae t cos(1t   ) (4.114)

Of the three solutions given by Eqs. (4.110), (4.112) and (4.114), we shall
concentrate on Eq. (4.114). It may be pointed out that the constants A1

and A2 in Eq. (4.110) are complex quantities, while B and C in Eq.

(4.112) and A and  in Eq. (4.114) are all real quantities.

The solution given by Eq. (4.114) indicate that for a damped oscillator
the motion is oscillatory due to the existence of the cosine function, but
the amplitude of the oscillation decays exponentially as shown in Fig.
4.7. The natural angular frequency, 1 , or the frequency of the damped

oscillator is always less than the free oscillation frequency 0 . The

natural frequency 1 is not a frequency in the true sense of the word


because the oscillator never passes through the same point twice with the
same velocity; that is the motion is not periodic. But if  is very small,

then 1  0 and we call 1 the “frequency”. If  is small, we can

expand Eq. (4.109) (using the binomial expansion) as

95
96

1
 2 
1     
1 2
2 2 2
 0 1  2 
 0 
0

 2 
 0 1    (4.115)
 20
2

 2 
 0 1  
2 
 20 

If   0 , then

1  0

Fig. 4.7: Motion of damped harmonic oscillator

Case (II) Critically damped, 02   2 : For this case, the two roots given

by Eqs. (4.105) and (106) are equal, that is

1  2   (4.116)

and the general solution given by Eq. (4.108) takes the form

x(t )   A1  A2 et  B1et (4.117)

where ( A1  A2 )  B1 =constant. This is not a general solution because it

contains only one constant. In such cases, we show that if e  t is a


solution, then

96
CLASSICAL MECHANICS

x  tet (4.118)

is also a solution. Substituting Eq. (4.118) into the differentia equation


(Eq. (4.99)), we get

 2
0 
  2 et  0 (4.119)

Since 0    , the equation is satisfied, and te  t is also a solution.

Thus, for a critically damped case, the general solution is a linear


combination of e  t and te  t ; that is

x(t )  B1  B2t e t (4.120)

where B1 and B2 are constants to be determined by the initial conditions.

Case (III) Overdamped (Strongly damped), 02   2 : If the damping

increases such that  2  02 , then the roots 1 and 2 are real. If we

represent

 2
 02 
1
2
 2 (4.121)

the general solution given by Eq. (4.108) takes the form


x(t )  et A1e 2 t  A2e 2 t  (4.122)

Note that 2 is no longer a frequency because the motion is not

oscillatory. Due to the decay term e  t multiplying both terms in the


parenthesis, the terms decay exponentially, one faster than the other and
become zero when sufficient time has elapsed. This is called strong,
over-damping or heavy damping. Once displaced or released, the body or
particle merely return to the equilibrium position although it may
overshoot. Fig. 4.8 shows the behavior of underdamped, overdamped
and critically damped harmonic oscillators from their equilibrium
positions.

97
98

Fig. 4.8: Return of harmonic oscillator to equilibrium

(a) Underdamped. (b) Overdamped. (c) Critically damped.

4.7 Energy Dissipation in Damped Motion


The total energy E (t ) of a damped harmonic system at any time t is
given by

E(t )  E(0)  Wf (4.123)

where E (0) is the total energy at time t  0 and W f is the work done

by friction in the time interval 0 to t . Assuming the dissipative


frictional force f  bx  bv , we can calculate W f as follows:

1
dx
W f   f dx   f dt   f v dt    b v 2 dt (4.124)
dt 0

Thus the rate of energy loss by friction may be written as

dE  dW f 
    bv2 (4.125)
dt  dt 

98
CLASSICAL MECHANICS

which is negative and represents the rate at which energy is dissipated


into heat. Since Wf  0 , the energy at any time Et continuously

decreases with time and may be calculated in the following manner:

1 2 1 2
E (t )  K (t )  U (t )  mx  kx (4.126)
2 2

From Eq. (4.114)

x(t )  Ae t cos(1t   )

and

  
x (t )  1 Ae t sin(1t   )  cos(1t   ) (4.127)
 1 

We shall now assume that the system is lightly damped so that


 / 1  1, and neglecting the second term on the right in the preceding
expression for x , we can substitute for x and x in Eq. (4.126) to obtain

E (t ) 
1 2  2t
2

A e m12 sin 2 (1t   )  k cos2 (1t   )  (4.128)

Since we assumed light damping, we may write 12  02  k / m ; hence

the equation takes the form

1 2  2t
E (t )  kA e (4.129)
2

while the initial energy of the system is obtained by substituting t  0 in


Eq. (4.129); that is

1 2
E0  kA (4.130)
2

Thus

E ( t )  E 0 e  2 t (4.131)

99
100

We see from this equation that the energy decreases ( or decays)


  than the rate at which the
exponentially at a much faster rate e 2t

 
amplitude decreases or decays et .

The time  in which E decreases to 1 / e ( ( 0.368) of its initial value


is called the characteristic time or decay constant and may be evaluated
by substituting E (t )  E0 / e and t   in Eq. (4.131):

E0
 E0e  2 (4.132)
e

or

2  1 (4.133)

That is,

1 2m m
   (4.134)
2 2b b

If  is small,    , and if  is very large,   0 .

We shall end our study of damped harmonic systems here. Another very
important and interesting system is forced harmonic oscillator (or driven
harmonic oscillator). Because of lack of time, you are asked to go and
read on this topic.

4.8 Summary
The harmonic oscillator in its various forms is one of the most important
problems in Physics. In this unit, the learner has been introduced to the
treatment of this problem for the case of free motion and three degrees of
damping. These topics should serve as a springboard for further study of
the problem in harmonic motion. Further problems include the driven
oscillator, the anharmonic oscillator and the oscillator damped by a force
with non-linear dependence of velocity.

100
CLASSICAL MECHANICS

4.8 Exercises
4.0 A block of mass m lying on a horizontal frictionless
surface is attached to two identical springs of spring
constant k as shown below.

The mass is displaced to a position x to the right and then released.

(i) Write down the equation of motion which describes the


motion of the mass at any time t .

(ii) Solve the equation of motion to obtain an expression of the


position of the mass at any time t .

(iii) Show that your solution found in (ii) above can also be
written as

B sin  0 t  C sin  0 t

where B and C are constants.

(iv) Obtain the relationship between the constants A and  to


your solution in (ii) and the new constant B and C in (iii)
above.
(v) Write down the expression of the natural frequency in terms
of k and the mass m .
(vi) Will the new natural frequency be higher or lower if one of
the springs is removed?
(vii) Explain two methods which you may use to damp the mo-
tion of the mass

101
102

4.1 The springs of a car of mass 1,200 kg give the car a vertical

oscillatory period of 0.5 s when the car is empty. How far does the car
sink when the driver and three passengers, each of mass 75 kg, get in
the car?

4.2 A weakly damped harmonic oscillator is at the point x0 and is at

rest at t  0 . What is its position as a function of time?

4.3 A particle of mass m moves under the conservative force with


potential energy

kx
V ( x) 
x  a2
2

where a and k are positive constants. Obtain the frequency of small


oscillations about the position of stable equilibrium.

4.4 A particle of mass m moves in a circular orbit under the action


of an attractive inverse-square central force


F (r )  
r2

(i) Prove that if the particle experiences a small radial disturbance,


then the periodic time of the perturbations is equal to the period
of revolution in the circular orbit.

(ii) If  is reduced instantaneously by half, then the particle will


subsequently move in a parabolic orbit.

102

You might also like